Sunteți pe pagina 1din 32

Distancia

Test 2VPediatría
Grupo CTO
CTO Medicina

Preguntas

1. Señale la afirmación INCORRECTA: 1) 5


1) Un niño cuadruplica su peso del nacimiento a los 2 años 2) 4
de vida y duplica su talla a los 4 años. 3) 3
2) La fontanela anterior suele cerrarse a los 12 meses y la 4) 2
posterior a las 6-8 semanas. 5) 1
3) El perímetro cefálico suele igualar al perímetro torácico al año.
4) La piel del RNPT está cubierta por una crema blanquecina 5. Señale la afirmación correcta respecto al test de Apgar:
llamada vérnix. 1) Se debe practicar únicamente a los RN patológicos o su-
5) Las lesiones dérmicas como el millium y el eritema tóxico puestamente patológicos.
carecen de significado patológico. 2) Entre los parámetros que se valoran se encuentra la frecuencia
respiratoria.
2. Todo lo siguiente es normal en un RN, EXCEPTO: 3) Se debe realizar al minuto y a los 5 minutos de vida.
1) Cierre del conducto de Arancio y del ductus. 4) Un test de Apgar de 4 al minuto de vida implica un alto
2) Eliminación de meconio en las primeras 48 horas de vida. riesgo de parálisis cerebral.
3) Menor capacidad para concentrar la orina que a edades 5) La palidez cutánea generalizada supone una puntuación de
posteriores. 1 en el test de Apgar.
4) FC de 140 lpm y FR de 30-40 rpm.
5) Hemoglobina más alta y leucocitos más bajos que en un 6. Con respecto al cefalohematoma, es FALSO que:
niño de 2 años. 1) Suele afectar a parietales con o sin fractura lineal asociada.
2) Es una hemorragia subperióstica cuya intensidad en ocasiones
3. Recién nacido a término que presenta a las pocas horas de vida puede requerir transfusión y/o fototerapia.
cianosis e intenso distress respiratorio. A la exploración, usted 3) Suele sobrepasar la línea media y líneas de sutura.
observa un abdomen excavado y ausculta el latido cardíaco 4) La piel suprayacente suele ser normal.
en el lado derecho. Respecto a la patología que sospecha, 5) La colección sanguínea aparece lentamente, por ello no se
señale la afirmación FALSA: hace manifiesto hasta varias horas postparto.
1) Se produce por alteración en el cierre de los canales pleu-
roperitoneales posterolaterales. 7. Recién nacido mediante un parto de nalgas presenta la mano
2) El diagnóstico suele hacerse por la radiografía, apreciándose derecha caída. Los dedos de dicha mano aparecen semiabier-
asas intestinales en el tórax y desplazamiento del mediastino. tos y no es capaz de agarrar el dedo de su madre con esa
3) El cierre del defecto anatómico es considerado una urgencia mano pero sí con la izquierda. Señale la respuesta incorrecta
quirúrgica. respecto a la lesión obstétrica que sufre este recién nacido:
4) Con frecuencia se asocia a otras malformaciones congénitas 1) Se debe a una lesión en las raíces inferiores del plexo braquial.
importantes. 2) El reflejo de Moro no estará presente en el lado derecho.
5) Entre las potenciales complicaciones graves destaca el de- 3) El parto de nalgas favorece esta situación.
sarrollo de hipertensión pulmonar y la hipoplasia pulmonar. 4) Puede asociarse con síndrome de Horner.
5) Este tipo de parálisis es menos frecuente que la parálisis de Erb.
4. Un recién nacido, a los 5 minutos de vida presenta: FC 60 latidos
por minuto, cianosis acra, esfuerzo respiratorio ausente, tono 8. Recién nacido de 42+4 semanas de edad gestacional y 4.250 g
muscular inexistente y ausencia de respuesta al introducir un de peso, nacido de parto de nalgas, que presenta en la explo-
catéter por su nariz. La puntuación de Apgar en este niño será: ración caput grande, crepitación a nivel de clavícula derecha,

CTO Medicina • C/Francisco Silvela, 106 • 28002 Madrid • Tfno.: (0034) 91 782 43 30/33/34 • E-mail: secretaria@ctomedicina.com • www.grupocto.es
1
Pediatría
Test 2V Distancia

mano derecha caída con reflejo de prensión palmar negativo,


Preguntas
Grupo CTO
CTO Medicina

aplicación de oxigenoterapia mediante presión parcial positiva


que a las 36 horas de vida sufre un empeoramiento de su espiratoria (PPPE). Al 3er día de vida, se produce mejoría clínica
estado general, con polipnea e hipotensión arterial. ¿Cuál de y gasométrica, permitiendo retirar la PPPE. Al 5º día de vida,
los siguientes datos NO esperaría encontrar en este niño? empeora la función respiratoria y se evidencia en la gasometría
1) Masa palpable en un costado. una retención de CO2. En la exploración destaca la ausencia
2) Anemia en el hemograma. de cianosis, pulsos periféricos saltones, frémito palpable en
3) Cociente neutrófilos inmaduros/neutrófilos totales mayor 2º espacio intercostal izquierdo. AC: soplo áspero en sístole y
de 0,16. diástole. ¿Cuál sería el diagnóstico más probable?
4) Ictericia. 1) Transposición de las grandes arterias.
5) Disminución del sodio y aumento del potasio. 2) CIV.
3) Tetralogía de Fallot.
9. RNPT de 3 días de vida y 2300 g de peso, que presenta en la 4) DAP.
exploración una tumoración abdominal cubierta por piel, en la 5) Coartación de aorta.
base del cordón umbilical, fácilmente reductible, palpándose
entonces un orificio herniario rodeado de bordes duros. ¿Cuál 13. RN de 44 semanas de edad gestacional, con antecedentes de
sería el diagnóstico más probable? sufrimiento fetal agudo, que en la 2ª hora de vida presenta
1) Onfalocele. distress respiratorio con mayor tendencia a pausas de apnea
2) Granuloma umbilical. y tórax abombado, que mejora tras 48 horas con presión
3) Gastrosquisis. parcial positiva al final de la espiración, pudiéndose retirar
4) Persistencia del uraco. ésta. Una vez en planta, presenta irritabilidad, inquietud y
5) Hernia umbilical. apnea, destacando en la exploración una hiperresonancia en
hemitórax derecho, con disminución del murmullo vesicular
10. RN de 36 semanas de EG, presenta, a las dos horas de vida, en dicho hemitórax. Lo más probable es que se trate de:
polipnea de 60 rpm, retracción subesternal y subcostal con 1) Enfermedad de membrana hialina, que ha evolucionado a
quejido espiratorio. Precisa administración de oxígeno, alcan- displasia broncopulmonar.
zando una FIO máxima de 0,35; con lo que mejora el cuadro 2) Taquipnea transitoria del RN complicada.
tras 6 horas de evolución. Respecto a este cuadro, señale la 3) Neumonía por Estreptococo del grupo B.
afirmación que NO es cierta: 4) Síndrome de aspiración meconial asociado a neumotórax
1) Entre sus factores predisponentes destaca un parto vaginal derecho.
rápido o por cesárea, así como la sobrehidratación materna. 5) Síndrome de Wilson-Mikity.
2) Se trata de un Síndrome de Avery, o también conocido como
distress respiratorio tipo II. 14. RNPT de 29 semanas de edad gestacional, que a los 31 días
3) Los pacientes suelen recuperarse alrededor de las 2 semanas de vida precisa oxígeno para mantener una saturación de
de edad. oxígeno adecuada. Con respecto al cuadro que presenta este
4) En la Rx de tórax se observan marcas vasculares prominentes paciente, señale la respuesta FALSA:
con líquido en las cisuras, hiperinsuflación y, ocasionalmente, 1) Si aparecen hepatomegalia y edemas en miembros, se debe
derrame pleural. sospechar una insuficiencia cardíaca derecha.
5) Sus rasgos distintivos son su rápida recuperación y el patrón 2) En el tratamiento es indispensable aumentar el aporte de
radiológico. líquidos, que mejora la evolución del cuadro.
3) La radiografía de este niño mostrará un patrón en esponja
11. El síndrome de distrés respiratorio tipo I o enfermedad de característico.
membrana hialina es responsable de forma directa o indirecta 4) Si no se encuentran antecedentes de EMH, se trata de un
del 50% de muertes en el período neonatal. Con respecto a síndrome de Wilson-Mikity.
ella, señale la aseveración FALSA: 5) Habitualmente se alcanza la normalidad desde el punto de
1) Se manifiesta como un cuadro de distress respiratorio precoz vista respiratorio en los primeros dos años.
que, a menudo, es relativamente refractario a la adminis-
tración de oxígeno. 15. Un recién nacido presenta taquipnea y en la radiografía de
2) En el 75-80% de los casos, al estudiar el líquido amniótico tórax se observan imágenes vasculares pulmonares promi-
deglutido, presenta un cociente L/E menor de 2 y ausencia nentes, diafragma aplanado y líquido en las cisuras. No existe
de fosfatidilglicerol. hipoxemia, hipercapnia o acidosis. ¿Cuál de las siguientes
3) Se caracteriza por un cuadro de hipercapnia progresiva con enfermedades es más probable?
leve hipoxia y acidosis metabólica. 1) Enfermedad de la membrana hialina.
4) En la Rx de tórax, es característica la presencia de un patrón 2) Aspiración de meconio.
reticulogranular con broncograma aéreo y menor volumen 3) Neumomediastino.
pulmonar. 4) Taquipnea transitoria del recién nacido.
5) La administración de surfactante intratraqueal en las primeras 5) Síndrome de Wilson-Mikity.
24 h disminuye la mortalidad de la enfermedad de membrana
hialina, pero no la incidencia de displasia broncopulmonar. 16. ¿Cuál de los siguientes recursos es el más adecuado para
tratar la hiperbilirrubinemia (11,2 mg/dl) en un lactante de 3
12. RN pretérmino, que al nacer presentó un cuadro de distress semanas, alimentado al pecho y con crecimiento y desarrollo
respiratorio, que precisó ingreso en cuidados intensivos y normales?

CTO Medicina • C/Francisco Silvela, 106 • 28002 Madrid • Tfno.: (0034) 91 782 43 30/33/34 • E-mail: secretaria@ctomedicina.com • www.grupocto.es
2
1) La fototerapia.
Preguntas
Grupo CTO
CTO Medicina
Test 2V
Distancia

4)
Pediatría

La supresión de la alimentación oral y el reposo intestinal


2) La transfusión de recambio. con descompresión mediante SNG es necesario.
3) El fenobarbital. 5) Se han involucrado en su etiología a diversos patógenos
4) Retirar definitivamente la lactancia materna. como E. coli, C. perfringens, Rotavirus y S. epidermidis.
5) Continuar con lactancia materna y tranquilizar a los padres.
21. Todos los siguientes grupos de recién nacidos tienen riesgo
17. En un RNPT de 30 semanas, intubado por EMH y con trata- de hipoglucemia neonatal salvo uno, señálelo:
miento antibiótico de amplio espectro por sospecha de sepsis, 1) Hijos de madre diabética.
a las 17 horas de vida se encuentran cifras de bilirrubina total 2) Niños con convulsiones neonatales.
de 20 mg/dl, con predominio de indirecta. Señale la opción 3) Neonatos con crecimiento intrauterino retardado y prema-
verdadera: turos.
1) El tratamiento con fototerapia simple exclusivamente sería 4) Enfermedades metabólicas como la galactosemia o las glu-
suficiente, pues es rápida y eficaz para este enfermo. cogenosis.
2) En este caso sería conveniente valorar realizar una exangui- 5) Neonatos gravemente enfermos, independientemente de
notransfusión, ante el elevado riesgo de neurotoxicidad. la patología de base.
3) El tratamiento con fenobarbital es efectivo en pocas horas,
aumentando la conjugación y excreción de bilirrubina. 22. RN, hijo de madre ADVP, comienza, a las 36 horas de vida,
4) No es necesario tomar ninguna medida, pues las cifras de con irritabilidad, temblores y mala tolerancia. Todas las si-
bilirrubina están en rango normal. guientes afirmaciones son ciertas, EXCEPTO:
5) Al ser un RNPT, tiene menos riesgo de kernicterus que un 1) Tienen mayor riesgo de desarrollar anomalías congénitas.
RNT. 2) La clínica del síndrome de abstinencia comienza antes en
hijos de madre adictas a heroína que a metadona.
18. Neonato de 2 días de vida, que presenta ictericia cutáneo- 3) Los hijos de adictas a metadona tienen más riesgo de sufrir
mucosa significativa. ¿Cuál de las siguientes opciones, con convulsiones.
respecto a las posibles causas de su ictericia, es FALSA? 4) Está indicado tratamiento con morfina oral.
1) Un parto traumático y un RN con múltiples hematomas a 5) La incidencia de EMH e hiperbilirrubinemia en RNBP de
la exploración, explican la ictericia. madres adictas a opiáceos es menor que en la población
2) Entre las causas más frecuentes de ictericia precoz, hay que general.
descartar un hipotiroidismo congénito.
3) El antecedente de bolsa rota durante 37 horas debe hacernos 23. Con respecto al hijo de madre diabética, NO es cierto que:
sospechar una posible sepsis. 1) Puede expresarse clínicamente retraso de la evacuación de
4) Una infección connatal por CMV puede debutar así. meconio.
5) Si el test de Coombs directo es positivo, se trata de una 2) Si la madre sufre trastornos vasculares, el niño puede ser
enfermedad hemolítica. CIR.
3) El 75% de los hijos de madre diabética desarrollarán después
19. RN de 32 semanas, con expulsión de meconio en las prime- hipoglucemia sintomática.
ras 24 horas, que en el 10º día de vida presenta intolerancia 4) Tienen mayor incidencia de distress respiratorio.
alimentaria, vómitos biliosos, distensión abdominal y una 5) La hipoplasia de colon izquierdo es una malformación
deposición sanguinolenta. En la analítica: neutropenia y intestinal a tener en cuenta en estos niños.
trombopenia con hiponatremia grave; pH 7,19; en la Rx de
tórax se observa neumoperitoneo. ¿Cuál sería el tratamiento 24. Sobre la etiología de las sepsis neonatales, señale la opción
más adecuado? correcta:
1) Descompresión nasogástrica, alimentación parenteral y 1) Candida albicans es un patógeno frecuente en la sepsis
antibioterapia sistémica. precoz de hijos de madres diabéticas.
2) Piloromiotomía extramucosa. 2) Staphylococcus epidermidis es el agente más frecuentemente
3) Enema de suero salino isotónico. responsable de sepsis tardía.
4) Enema de contraste yodado. 3) Streptococcus agalactiae serotipo III y E. coli serotipo K1 son
5) Resección del intestino necrosado y colostomía de descarga. los agentes que más frecuentemente producen meningitis
neonatal.
20. Un RN pretérmino, con antecedentes de sufrimiento fetal, 4) Strepcoccus pneumoniae es el patógeno más frecuente en
comienza con distensión abdominal importante, vómitos y la sepsis precoz en nuestro medio.
deposiciones hemorrágicas. En la Rx de abdomen se observa 5) Si se sospecha Listeria monocytogenes, se debe emplear
edema de asas intestinales, con un patrón en “miga de pan” cefotaxima en vez de gentamicina.
y presencia de gas en la pared intestinal. Respecto a esta pa-
tología, señale cuál de las siguientes afirmaciones es FALSA: 25. RN de 35 semanas, con antecedente de bolsa rota de más de
1) Las situaciones de hipoxia y bajo gasto predisponen a esta 24 horas, presenta, a las 8 horas de vida, hipotermia, mala
patología. perfusión periférica, letargia y taquipnea. En la analítica des-
2) La primera manifestación clínica es la aparición de distensión taca leucocitos 32.430, con índice infeccioso de 0,32; señale
abdominal. lo FALSO sobre la sepsis del recién nacido:
3) La presencia de neumatosis intestinal es indicación de cirugía 1) Las manifestaciones neurológicas predominan en la sepsis
urgente. de inicio precoz.

CTO Medicina • C/Francisco Silvela, 106 • 28002 Madrid • Tfno.: (0034) 91 782 43 30/33/34 • E-mail: secretaria@ctomedicina.com • www.grupocto.es
3
Pediatría

2)
Test 2V Distancia

Los agentes causales más frecuentes son el estreptococo del


Preguntas
Grupo CTO
CTO Medicina

30. RN de madre 0- y padre A+, con embarazo no controlado.


grupo B y E. coli. Con respecto a los cuadros clínicos que puede presentar el
3) Se debe sospechar en todo neonato con temperatura ines- neonato, señale la opción FALSA:
table y rechazo del alimento. 1) Si el Coombs indirecto para la incompatibilidad Rh es
4) La mortalidad es mayor en aquella que aparece antes de la negativo, se debe realizar profilaxis con gammaglobulina
semana de vida. hacia la semana 28 de gestación, y en las primeras 72 horas
5) El tratamiento empírico adecuado es ampicilina y gentami- postparto, si el RN es Rh+.
cina, durante al menos 7 días. 2) Puede ser necesaria la transfusión intrauterina de hematíes,
si la hemólisis es intensa.
26. ¿Cuál sería la pauta más correcta a llevar a cabo en el RN de 3) En la incompatibilidad de grupo puede haber afectación
madre con la siguiente serología: Ag s + frente al virus de la del primer hijo.
hepatitis B? 4) Si el Coombs directo es positivo frente al Ag Rh, es impres-
1) Vacuna. cindible administrar gammaglobulina a la madre.
2) Gammaglobulina. 5) La aparición de una DO 450 en zona III en el análisis es-
3) Interferón Alfa. pectrofotométrico del líquido amniótico indica un riesgo
4) Vacuna + Gammablobulina. fetal alto.
5) Vacuna+ gammaglobulina+ interferón.
31. Señale la opción FALSA, con respecto a la policitemia neonatal:
27. Señale la afirmación INCORRECTA, respecto a las alteracio- 1) Puede ser causa de ictericia en la primera semana de vida.
nes endocrinológicas que pueden detectarse en el período 2) Es más frecuente en los hijos de madre diabética.
neonatal: 3) Se diagnostica con un hematocrito capilar > 65%.
1) El panhipopituitarismo puede manifestarse como micropene 4) Puede complicarse con insuficiencia cardíaca.
e hipoglucemia. 5) Es más frecuente en los RN con bajo peso para su edad
2) El hipoparatiroidismo puede manifestarse clínicamente como gestacional por insuficiencia placentaria.
tetania.
3) La hemorragia adrenal aguda puede verse tras partos de 32. Un RNPT de bajo peso al nacer, presenta un cuadro com-
nalgas, partos traumáticos o infecciones fulminantes. patible con una crisis motora focal. En la TC craneal apa-
4) El RN con hipopituitarismo o déficit aislado de GH suele recen ventrículos ligeramente dilatados y calcificaciones
tener una talla y un peso al nacimiento menor de -2DS de periventriculares. ¿Qué patología es más probable que
la media. presente este RN?
5) La insuficiencia suprarrenal se puede manifestar por vómitos, 1) Sífilis congénita.
diarrea, deshidratación, hiperpotasemia e hiponatremia. 2) Toxoplasmosis congénita.
3) Rubéola congénita.
28. RNT, sin antecedentes de interés, al que se le realizó profilaxis 4) Varicela congénita.
ocular y antihemorrágica a los pocos minutos del nacimien- 5) Infección prenatal por citomegalovirus.
to, presenta, a las 48 horas de vida, sangrado por el cordón
umbilical y sangre fresca en heces. Señale la INCORRECTA: 33. La premadurez es cada vez más frecuente y se asocia a un
1) No estará indicado administrar vitamina K, porque ya ha aumento de la morbimortalidad. ¿Cuál de las siguientes res-
recibido una dosis de profilaxis. puestas es correcta acerca de los cuidados específicos que
2) Un factor de riesgo para la aparición de este cuadro es el deben recibir estos recién nacidos?
tratamiento materno con fenitoína. 1) Es recomendable introducirles en incubadora, a una tem-
3) Estaría indicado realizar un estudio de coagulación. peratura siempre fija de 36,5-37 oC, coincidiendo con la
4) El cuadro puede deberse a una sepsis con CID. temperatura corporal materna.
5) La enfermedad hemorrágica por déficit de vitamina K de 2) La alimentación debe iniciarse por sonda nasogástrica en
comienzo tardío (>1 semana) puede estar asociada a una todos los menores de 32 semanas de edad gestacional.
hepatitis neonatal o atresia biliar. 3) La oxigenoterapia es una práctica rutinaria en los prematuros,
por el riesgo de desarrollar un cuadro de distrés respiratorio
29. Con respecto a la anemia en el período neonatal, NO es cierto en las primeras 72 horas de vida.
que: 4) La leche materna es la más adecuada, añadiéndose suple-
1) La cifra de hemoglobina en el RNT al nacimiento es aproxi- mentos de proteínas, calcio y fósforo.
madamente de 14-20 g/dl y en el RNPT, 1-2 g menos. 5) En los prematuros que necesitan fluidoterapia intravenosa,
2) Una de las causas más frecuentes de anemia al nacer es la hay que tener en cuenta que las pérdidas insensibles son
enfermedad hemolítica. menores que en los nacidos a término, por lo que los aportes
3) Una transfusión fetomaterna puede diagnosticarse por la líquidos deben ser menores.
prueba de Kleihauer-Betke en la sangre materna.
4) Una pérdida aguda de sangre inicialmente puede cursar con 34. Recién nacido a término, que traen a Urgencias sus padres
hemoglobina normal y sin hepatoesplenomegalia, pero con a los 11 días de vida, por “enrojecimiento y supuración” del
signos incipientes de shock. ombligo, que han detectado ese mismo día. El niño no ha
5) Existe un descenso fisiológico en las cifras de Hb, que se tenido fiebre, tiene buen estado general y come bien, según
produce en la 6ª semana de vida en los RNT y hacia la 8-12ª refieren. En la exploración física destaca una tumoración
semana en los RNPT. ligeramente prominente en el ombligo, rosada, blanda y

CTO Medicina • C/Francisco Silvela, 106 • 28002 Madrid • Tfno.: (0034) 91 782 43 30/33/34 • E-mail: secretaria@ctomedicina.com • www.grupocto.es
4
Preguntas
Grupo CTO
CTO Medicina

granular a la palpación, con secreción serosa y no purulenta,


Test 2V
Distancia

4)
Pediatría

La enfermedad hemorrágica del RN es más frecuente en los


con normalidad de la piel de alrededor. ¿Cuál sería su actitud niños que se alimentan con fórmula artificial.
diagnóstico-terapeútica? 5) Los ácidos grasos poliinsaturados existen en mayor concen-
1) Realizar hemograma, PCR y recoger muestra para hemo- tración en la leche de madre.
cultivo, iniciando antibioterapia hasta el resultado de éste.
2) Realizar biopsia y decidir el tratamiento una vez tenga el 39. Acude a su consulta una madre con su hija de 10 años preocu-
informe anatomopatológico. pada porque dice que su hija “siempre ha sido la más bajita
3) Cauterizar con nitrato de plata en varias sesiones hasta llegar de toda la clase”. Su talla actual se encuentra en el percentil 2
a la base. de la población general. Su talla genética está situada en el
4) Esperar hasta el año de edad, porque lo más probable es percentil 3. Su desarrollo sexual es el que corresponde para
que la formación vaya desapareciendo espontáneamente su edad y su edad ósea es de 9 años. ¿Qué situación cree
hasta entonces. usted que presenta la niña?
5) Extirpación quirúrgica completa, por el riesgo de degene- 1) Un hipocrecimiento por deficiencia de la hormona de
ración maligna. crecimiento.
2) Un hipocrecimiento por un síndrome de Turner.
35. Señale la afirmación FALSA, de entre las siguientes, respecto 3) Un hipocrecimiento por un hipotiroidismo congénito.
a la ictericia fisiológica: 4) Un retraso constitucional del crecimiento y del desarrollo.
1) Las cifras máximas de bilirrubina suelen alcanzarse a los 3-4 5) Una talla baja familiar.
días.
2) La ictericia en los RNPT suele ser de inicio algo más tardío 40. Niño de 4 meses y 5 kg de peso, que presenta deposiciones
y más prolongada que en los RNT. líquidas frecuentes (15/día), desde hace 72 horas, con febrí-
3) Puede comenzar en las primeras 24 horas de vida en el RNT. cula y mala tolerancia oral. Peso 4,5 kg, TA: 75/60. Sequedad
4) Su duración suele ser inferior a 10-15 días. de piel y mucosas, irritabilidad, pulsos palpables. Resto de
5) En los RNPT pueden alcanzarse cifras de hasta 14 mg/dl. exploración normal. La analítica muestra: Na+: 170 mEq/l;
K+: 4,0 mEq/l; Cl-: 100 mEq/l; pH 7,10; bicarbonato 9 mEq/l
36. RN a término al que se le determina el valor de TSH en san- y PCO2: 18. Señale la opción correcta:
gre, obtenida mediante la prueba del talón, el día 3 de vida. 1) Reposición rápida con suero salino fisiológico a 20 cc/kg,
El valor de la TSH aparece elevado. ¿Qué actitud considera ante la situación de shock de este paciente.
más adecuada?: 2) Corrección lenta en 72 horas, debido al riesgo neuroló-
1) Repetir la prueba dentro de un mes. gico.
2) Sospechar hipertiroidismo e iniciar tratamiento con antiti- 3) Corrección con suero glucosado en 24 horas.
roideos. 4) Serán llamativos los signos de depleción vascular.
3) Tranquilizar a la madre porque se trata de algo habitual en 5) Es la forma de deshidratación más frecuente.
los RN.
4) Sospechar hipotiroidismo congénito e iniciar tratamiento 41. Señale cuál de las siguientes afirmaciones sobre el raquitismo
sustitutivo con hormona tiroidea. NO es cierta:
5) Ingresar al RN en observación. 1) La existencia de craneotabes parietal en un RN debe ha-
cernos sospechar la existencia de un raquitismo por déficit
37. Niña de 20 días de vida, que acude por presentar dificultad de vitamina D.
para la alimentación, con inactividad, apatía y escaso llanto, 2) Las alteraciones radiológicas preceden en la mayoría de los
así como respiración ruidosa. Entre sus antecedentes destaca casos a las manifestaciones clínicas.
haber precisado ingreso hospitalario por ictericia. En la ex- 3) Las necesidades diarias de vitamina D en un RNT son de
ploración presenta apatía, abdomen voluminoso, con hernia 400 UI.
umbilical, edema de miembros con piel fría y moteada. La 4) En la Rx se observan metáfisis ensanchadas con imagen “en
fontanela anterior mide 3x3 cm y la posterior 2x2. Presenta copa”.
baja implantación del pelo y macroglosia. El recuento y fór- 5) El raquitismo por déficit de vitamina D es más frecuente en
mula sanguínea son normales, sin alteraciones en los datos niños de raza negra.
bioquímicos. El diagnóstico más probable es:
1) Hipertiroidismo congénito. 42. Las variantes de la normalidad son las causas más frecuentes
2) Fenilcetonuria. de talla por debajo de 2 desviaciones estándar. Con respecto
3) Hipotiroidismo congénito. a ellas, NO es cierto que:
4) Raquitismo. 1) Los niveles de hormona del crecimiento son normales.
5) Hiperplasia suprarrenal congénita por déficit de 21-hidroxi- 2) En la talla baja de origen genético, la edad ósea es menor
lasa. que la cronológica.
3) En el retraso constitucional del crecimiento, la edad ósea
38. Sobre las diferencias entre leche de vaca y leche materna, coincide con la estatural y ambas son menores que la cro-
señale la opción correcta: nológica.
1) La leche de madre es más rica en grasas que la de vaca. 4) El pronóstico de talla es normal o ligeramente disminuido
2) El flúor es más abundante en la leche de vaca. en el retraso constitucional.
3) Las seroproteínas se encuentran en mayor porcentaje en la 5) En la talla baja familiar, la pubertad aparece dentro de los
leche de vaca. límites de la normalidad.

CTO Medicina • C/Francisco Silvela, 106 • 28002 Madrid • Tfno.: (0034) 91 782 43 30/33/34 • E-mail: secretaria@ctomedicina.com • www.grupocto.es
5
Pediatría
Test 2V Distancia

43. Niño de 9 meses, bien vacunado, que a la una de la madru-


Preguntas
Grupo CTO
CTO Medicina

garganta. En la exploración se observa una masa en la pared


gada, en pleno invierno, acude a Urgencias por despertarse posterior de la faringe. El diagnóstico más probable sería:
de pronto con tos perruna, fiebre de 38 oC y agitación. No 1) Faringolaringitis aguda.
babea y toma algunos líquidos. En los dos días pasados ha 2) Cuerpo extraño.
tenido una infección respiratoria alta (como el resto de la 3) Epiglotitis aguda.
familia). ¿Cuál es el diagnóstico más probable? 4) Traqueítis bacteriana.
1) Epiglotitis por H. influenzae. 5) Absceso retrofaríngeo.
2) Crup viral.
3) Crup espasmódico. 48. Niño de 6 años que, estando previamente bien, inicia de
4) Difteria. forma progresiva una tos seca con dificultad respiratoria. En
5) Cuerpo extraño en laringe. la exploración está afebril, con ligera cianosis periorificial,
tiraje subcostal y supraesternal con sibilancias audibles a
44. Niño de 3 años de edad, atendido 3 días antes en Urgencias distancia. En la auscultación pulmonar presenta espiración
por un cuadro típico de laringotraqueítis vírica. La madre alargada con hipoventilación bilateral y sibilancias espiratorias
consulta de nuevo por empeoramiento de la tos y la fiebre, difusas. La Rx de tórax muestra hiperinsuflación pulmonar
decaimiento, y dificultad respiratoria en las últimas 24 horas. con costillas horizontalizadas y diafragma aplanado, así como
En la exploración física el niño tiene regular estado general, un ligero aumento de la trama broncovascular. El diagnóstico
con dificultad respiratoria llamativa y estridor inspiratorio más probable será:
y espiratorio. En la auscultación se oyen roncus dispersos. 1) Bronquiolitis.
Señale la respuesta correcta acerca del caso: 2) Tos psicógena.
1) Hay que sospechar una neumonía, siendo el antibiótico 3) Crisis asmática.
empírico indicado en esta edad la amoxicilina. 4) Neumonía por Mycoplasma spp.
2) Hay que administrar una dosis de salbutamol inhalado, y si 5) Crup espasmódico.
no mejora, realizar una radiografía de tórax.
3) Lo más importante es asegurar la permeabilidad de la vía 49. Kevin es un niño de cuatro años, que acude a urgencias del
aérea, con intubación endotraqueal hasta que sea efectivo hospital en el que Vd. está de guardia a las dos de la madru-
el tratamiento etiológico. gada. La madre refiere que desde la tarde se queja de dolor
4) El principal agente etiológico es el S. aureus. de garganta y presenta fiebre de 39,5 oC. Cuando Vd. lo ve,
5) Las complicaciones son relativamente frecuentes, siendo la está sentado, con la boca abierta, presentando salivación
más importante el desarrollo de bronquiectasias. intensa. Con respecto al proceso que Vd. sospecha, señale
la opción FALSA, de entre las siguientes:
45. ¿Cuál de los siguientes apoyaría el diagnóstico de crisis de 1) Es probable que Kevin no haya completado el calendario
broncoespasmo frente al de bronquiolitis en un lactante de vacunal.
11 meses que presenta febrícula, tos y dificultad respiratoria 2) Este cuadro clínico no suele precisar intubación endotraqueal.
desde hace 24 horas? 3) En la Rx lateral de faringe se podrá apreciar la epiglotis
1) Antecedentes personales de episodios similares. aumentada de tamaño.
2) Comienzo progresivo, con signos iniciales de infección de 4) El tratamiento farmacológico de elección es la cefotaxima i.v.
vías respiratorias altas. 5) Pueden aparecer como complicación otras infecciones,
3) Ausencia de mejoría tras la administración de broncodila- como la otitis media o la neumonía.
tadores nebulizados.
4) Madre fumadora. 50. El pilar del tratamiento de la fibrosis quística es la terapia
5) Signos de hiperinsuflación pulmonar en la radiografía de pulmonar, cuyo objetivo principal es eliminar las secreciones y
tórax. controlar las infecciones. Señale la que le parece INCORRECTA
de las siguientes afirmaciones:
46. Con respecto a la bronquiolitis aguda, es FALSO que: 1) La fisioterapia respiratoria está indicada de 1 a 4 veces
1) El adenovirus puede asociarse a complicaciones a largo plazo, al día.
como bronquiolitis obliterante y síndrome de hiperclaridad 2) La antibioterapia vía oral está indicada siempre que haya
pulmonar unilateral. signos o síntomas de exacerbación respiratoria, eligiéndose los
2) La fuente de la infección viral suele ser un miembro de la fármacos de forma empírica, pues la realización de cultivos
familia con una enfermedad respiratoria leve. en estos pacientes es poco rentable.
3) La exploración muestra un lactante taquipneico con dificultad 3) La tobramicina en aerosol consigue disminuir el número
respiratoria y sibilancias espiratorias. de hospitalizaciones de los pacientes colonizados por
4) Casi la mitad de los lactantes que padecieron bronquiolitis P. aeruginosa y mejorar su función pulmonar después de
presentará en un futuro hiperreactividad de las vías aéreas 6 meses de tratamiento.
durante la infancia. 4) Las dosis de antibióticos orales en estos pacientes suelen ser
5) Son raras las complicaciones bacterianas, tales como bron- 2-3 veces mayores que las convencionales.
coneumonía u OMA. 5) Los corticoides sistémicos se utilizan para tratar la aspergilosis
alérgica.
47. Niño de 6 años, diagnosticado de faringitis aguda hace 6
días, comienza de forma brusca con fiebre alta, dificultad 51. El primogénito de una pareja sufre en el periodo neonatal íleo
para tragar, intenso distrés respiratorio e importante dolor de meconial. ¿Cuál es el método de diagnóstico principal que

CTO Medicina • C/Francisco Silvela, 106 • 28002 Madrid • Tfno.: (0034) 91 782 43 30/33/34 • E-mail: secretaria@ctomedicina.com • www.grupocto.es
6
Preguntas
Grupo CTO
CTO Medicina

llevaría a cabo para establecer el diagnóstico en la enfermedad


Test 2V
Distancia

1)
Pediatría

Retirar el gluten de la dieta y después realizar biopsia intestinal.


subyacente más probable? 2) Mantener el gluten al menos dos años, y después hacer
1) Test del tripsinógeno inmunorreactivo. biopsia intestinal.
2) Test de meconio. 3) Solicitar los anticuerpos IgA-antitransglutaminasa y un re-
3) Estudio molecular genético directo. cuento de IgA sérica total
4) Test de cloro en sudor. 4) Retirar definitivamente el gluten de la dieta, sin necesidad
5) Estudio genético de vínculo o cercanía. de realizar ninguna prueba complementaria.
5) Instaurar dieta astringente y calmar a los padres.
52. Varón de 7 meses, que comienza de forma brusca con crisis de
llanto intenso, acompañadas de encogimiento de miembros 56. En la biopsia del intestino delgado de un paciente celíaco,
inferiores y palidez cutánea llamativa. Entre una crisis y otra, el encontraría:
niño permanece somnoliento y decaído. En la exploración física 1) Inclusiones PAS+.
aparece distensión abdominal y se palpa una masa en hipocondrio 2) Escaso infiltrado linfocitario.
derecho. Señale la afirmación FALSA sobre este cuadro: 3) Patrón digitiforme de la mucosa.
1) Es la causa más frecuente de obstrucción intestinal entre los 4) Atrofia subtotal de las vellosidades con hiperplasia de las
3 meses y 6 años. criptas.
2) Sólo en un 2-5% se ha podido encontrar una causa orgánica 5) Biopsia patognomónica.
que lo favorezca.
3) En la Rx de abdomen se puede encontrar un silencio aéreo 57. Señale la afirmación FALSA al hablar de la enfermedad
en hipocondrio y flanco derecho. celíaca:
4) La mortalidad aumenta si los síntomas están presentes du- 1) La lesión intestinal es mayor en la porción proximal del
rante más de 48 horas. intestino delgado.
5) La localización más frecuente es la exclusivamente ileal. 2) El 10% de los niños estudiados por retraso de crecimiento
son portadores.
53. Con respecto a la invaginación intestinal, señale la opción 3) La positividad de los Ac se relaciona con la actividad de la
FALSA: enfermedad.
1) Si este cuadro aparece en niños mayores de 6 años, debemos 4) Los Ac antigliadina son los más sensibles y específicos.
descartar un linfoma intestinal. 5) Los pacientes no tratados tienen más predisposición a pa-
2) Si no se diagnostica precozmente, aparecen las deposiciones decer linfomas en la edad adulta.
“en jalea de grosella”.
3) La técnica diagnóstica de elección es la ecografía abdominal. 58. Paciente varón, con síndrome de Down de dos meses y medio
4) Hasta un 45% de los casos tratados con enema recidivan. de edad, que refiere tos nocturna. La madre cuenta que, en
5) El signo del “muelle enrollado” nos ayuda a establecer el ocasiones, se pone muy rígido, con la cabeza hiperextendida.
diagnóstico. Estos episodios le ocurren sobre todo un rato después de las
tomas. Con respecto al cuadro que Vd. sospecha, señale la
54. Recién nacido que presenta inmediatamente tras el nacimiento opción FALSA:
exceso de secreciones orales con el abdomen llamativamente 1) En el hemograma podemos encontrar una anemia ferropé-
distendido y timpanizado. Ha nacido a la semana 35 de ges- nica.
tación tras cesárea por polihidramnios materno. ¿Cuál de las 2) Esta entidad es más frecuente en varones, y aparece entre
respuestas es INCORRECTA acerca de la patología que debe 1/300 y 1/1000 niños, de forma acentuada.
sospechar? 3) La prueba diagnóstica de elección es el esofagograma con
1) Para confirmar el diagnóstico lo más útil sería hacer una contraste de bario.
radiografía de tórax. 4) En los casos leves, puede no ser necesario el tratamiento
2) Son excepcionales las secuelas después del tratamiento farmacológico.
quirúrgico. 5) Puede debutar como neumonías de repetición por aspira-
3) El tratamiento constituye una urgencia quirúrgica, siendo ciones.
conveniente que el niño permanezca en decúbito prono
con aspiración gástrica hasta la intervención. 59. Varón de 7 meses, que acude a la consulta por estreñimien-
4) Es conveniente descartar cardiopatías estructurales asociadas to crónico, intermitente, desde el nacimiento, que ha sido
antes de la cirugía. resuelto con dificultad por la familia con distintas ayudas.
5) El 50% de los pacientes tienen otras anomalías congénitas La madre refiere que permanece sin hacer deposición hasta
asociadas. 1 semana y que ésta es de características caprinas. Entre los
antecedentes personales destaca expulsión de meconio a
55. Niña de 24 meses consulta por estancamiento de la curva las 72 horas de vida. De la exploración física cabe señalar:
ponderal. Desde hace unos meses presenta heces más blan- peso y talla en percentil 3, distensión abdominal moderada
das, pastosas, fétidas y pegajosas. Presenta mal carácter, y palpación de masa fecal en fosa ilíaca derecha. En el tacto
anorexia y decaimiento. No vómitos ni dolor abdominal. rectal no se encuentran heces en la ampolla rectal. Con
Se le realiza un hemograma con Hb: 10 g/dl, Fe sérico respecto al diagnóstico más probable de este paciente, es
8 mg/dl, ferritina 4 ng/ml. Test de Van de Kamer con grasas cierto que:
totales de 7 g/día. ¿Cuál sería la actitud más adecuada en 1) La existencia de diarrea en un caso similar permitiría excluir
este paciente? un megacolon congénito.

CTO Medicina • C/Francisco Silvela, 106 • 28002 Madrid • Tfno.: (0034) 91 782 43 30/33/34 • E-mail: secretaria@ctomedicina.com • www.grupocto.es
7
Pediatría

2)
Test 2V Distancia

La alteración más característica en el enema opaco es un


Preguntas
Grupo CTO
CTO Medicina

tres días. En la exploración se observa dudosa masa de 2 cm


megarrecto asociado a una dilatación del colon. en hipocondrio derecho. ¿Cuál es su diagnóstico de sospecha?
3) En la manometría anorrectal se detecta una relajación del 1) Hernia hiatal.
esfínter anal interno. 2) Mala técnica alimentaria.
4) El diagnóstico definitivo se realizaría mediante una biopsia 3) Insuficiencia suprarrenal.
rectal. 4) Estenosis hipertrófica de píloro.
5) El tratamiento de elección es dietético y mediante el uso de 5) Atresia duodenal.
medicamentos laxantes.
64. El dolor abdominal recurrente es un motivo de consulta fre-
60. Varón de 14 meses que, desde hace aproximadamente 2 meses, cuente en los niños de la etapa preescolar y escolar. Señale
realiza 4 deposiciones al día, líquidas, no malolientes ni brillan- la opción correcta acerca de esta entidad:
tes, y sin olor ácido, que no contienen sangre ni pus, pero en 1) En la mayoría de los menores de 2 años no se llega a iden-
las que, a simple vista, se observan restos vegetales sin digerir. tificar una causa orgánica.
El niño no ha perdido peso, ni se encuentra mal por culpa de la 2) Los casos en que el dolor es “funcional” son de mejor
diarrea. La exploración física es normal, con peso y talla en el pronóstico, mejorando por lo general en cuanto los es-
percentil 50, sin signos de deshidratación, distensión abdominal tudios se completan sin encontrarse ninguna patología.
ni eritema perianal. El estudio de heces y sangre es normal. ¿Cuál 3) La asociación del dolor a las comidas, y su presencia por
es la actitud que tomaría con este paciente? la noche, son dos patrones que indican casi con seguridad
1) Realizar biopsia intestinal, y si es patológica, retirar el gluten la existencia de una enfermedad orgánica.
de la dieta. 4) Salvo que los síntomas sugieran una enfermedad péptica,
2) Tranquilizar a los padres, y aconsejar una dieta sin limita- no está indicado realizar de rutina pruebas de detección de
ciones, adecuada para la edad del niño. H. pylori.
3) Realizar una curva de sobrecarga con lactosa, y si es pato- 5) Aunque la historia clínica y la exploración física indiquen
lógica, retirar el consumo de la misma. un origen funcional, es necesario siempre realizar pruebas
4) Investigar antecedentes familiares y personales de patología complementarias para descartar organicidad.
respiratoria y realizar test del sudor.
5) Instaurar dieta astringente, con restricción de leche y pro- 65. Niño de 2 años que, estando previamente bien, comienza con
ductos lácteos, y pobre en grasa. sangrado rectal, sin acompañarse de dolor. Para descartar un
divertículo de Meckel, se le realiza una gammagrafía. Señala
61. Niño de 2 meses, que ha comenzado con lactancia artificial la respuesta verdadera:
unas horas antes, y que actualmente presenta vómitos, dia- 1) La prueba más indicada en este paciente habría sido una
rrea líquida y leve hinchazón de labios y párpados, así como Rx de abdomen.
un exantema urticariforme en tronco y extremidades. Como 2) El estudio de contraste con bario es de gran utilidad.
único antecedente de interés en la familia, destaca el padre 3) La gammagrafía con Tc-99 es la técnica más sensible inde-
con asma extrínseca y atopia. La actitud terapéutica más pendientemente de que exista mucosa ectópica.
adecuada en este paciente sería: 4) El tratamiento en este paciente sería únicamente sintomático.
1) Sustituir su fórmula artificial normal por un preparado a base 5) La captación del isótopo aumenta si administramos previa-
de leche sin lactosa. mente cimetidina, glucagón o gastrina.
2) Continuar con su fórmula artificial y ver evolución.
3) Alternar unas tomas con su fórmula artificial y otras con un 66. ¿Cuál de las siguientes características es más frecuente y peculiar
hidrolizado de proteínas. de los vómitos por reflujo gastroesofágico (calasia) en el niño?
4) Reemplazar su fórmula artificial por una fórmula de soja. 1) Asociación con ferropenia.
5) Reemplazar su fórmula artificial por un hidrolizado proteico. 2) Relación con la ingesta.
3) Repercusión sobre el crecimiento.
62. Niño de 13 meses, que tras padecer una gastroenteritis aguda 4) Comienzo neonatal y mejoría con la edad.
hace una semana, presenta una diarrea líquida que produce 5) Asociación con síntomas de broncoconstricción.
eritema en la zona del pañal. Al explorar al niño advierte un
aumento de los ruidos hidroaéreos. Respecto a esta patología 67. Varón de 5 años que, desde hace una semana, presenta un
qué opción es incorrecta: cuadro de varicela. En la actualidad, la mayoría de las lesiones
1) El pH fecal suele ser menor de 7. están en fase de costra y se encuentra afebril. De forma brusca,
2) La prueba del Clinitest es positiva en la malabsorción de presenta vómitos persistentes, proyectivos, con deterioro del
hidratos de carbono del tipo lactosa. nivel de conciencia y alteraciones del comportamiento, vol-
3) La causa más frecuente de deficiencia secundaria de lactasa viéndose agresivo. En la exploración sólo llama la atención una
es la diarrea infecciosa. discreta hepatomegalia. En la analítica presenta hipoglucemia,
4) En la intolerancia secundaria a hidratos de carbono, la elevación importante de la GOT, GPT, LDH y CPK, aumento del
exclusión de disacáridos es permanente. amonio sérico. Señale cuál será el diagnóstico más probable:
5) Una nutrición precoz con dieta normal en pacientes que 1) Encefalitis postvaricelosa.
han sufrido una GEA evita su aparición. 2) Hepatitis A aguda.
3) Síndrome de Reye.
63. Niño de 3 semanas que presenta vómitos alimentarios, no 4) Encefalitis herpética.
biliosos y abundantes con la mayoría de las tomas, desde hace 5) Debut de una metabolopatía, pendiente de filiar.

CTO Medicina • C/Francisco Silvela, 106 • 28002 Madrid • Tfno.: (0034) 91 782 43 30/33/34 • E-mail: secretaria@ctomedicina.com • www.grupocto.es
8
68. Sobre el síndrome de Reye, es FALSO:
Preguntas
Grupo CTO
CTO Medicina
Test 2V
Distancia

73. La causa más frecuente de HTA en la infancia es:


Pediatría

1) Debe diferenciarse de algunas metabolopatías, como los 1) Coartación de aorta.


defectos de oxidación de ácidos grasos. 2) Hiperplasia suprarrenal congénita.
2) Su cuadro clínico se caracteriza por el desarrollo de focalidad 3) HTA esencial.
neurológica. 4) Nefropatía por reflujo.
3) Existe una disfunción mitocondrial subyacente. 5) Trombosis de la arteria renal.
4) Parece asociarse al uso de aspirina en las infecciones virales.
5) En el tratamiento, las medidas antiedema cerebral son 74. Un niño de 12 años y 5 meses se presenta en el Servicio
importantes. de Urgencias con dolor inguinal y escrotal intenso, que
comenzó de forma brusca después de un partido de fútbol.
69. Cuando se diagnostica de fimosis a un lactante, al comen- El niño está afectado y se observa tumefacción y dolor en
tarles sobre el proceso a sus padres tenemos que tener en hemiescroto derecho, no se detecta reflejo cremastérico
cuenta que: y el teste derecho está elevado y rotado. Señale la opción
1) Hasta los 2 años se puede tratar de un proceso fisiológico. FALSA, respecto a este cuadro:
2) Es un hallazgo poco frecuente. 1) Habitualmente ocurre en niños mayores de doce años.
3) Se realiza circuncisión solo en caso de existir motivos religiosos. 2) La torsión del apéndice testicular es más frecuente entre los
4) Las adherencias peneanas y la fimosis se trata de una misma 2 y los 11 años.
patología. 3) En niños mayores de 13 años debe hacerse el diagnóstico
5) En la actualidad el uso de pomadas de corticoides evita la diferencial con la epididimitis.
circuncisión por motivos médicos. 4) El tratamiento óptimo es la cirugía inmediata.
5) La torsión testicular puede producirse en el periodo neonatal,
70. Señale la respuesta FALSA sobre la estenosis pielocalicial pero en estos casos no suele haber ningún riesgo de perder
congénita: las gónadas.
1) Es la causa más frecuente de obstrucción de la vía urinaria
en la infancia. 75. Lactante varón de 10 meses al que en un examen rutinario de
2) Es dos veces más frecuentes en varones que en mujeres. salud se le descubre una masa en flanco izquierdo, dura, que
3) En el 10% de los casos la estenosis es bilateral. sobrepasa línea media. En la ecografía abdominal dicha masa
4) Los casos detectados prenatalmente como hidronefrosis se corresponde con un tumor sólido localizado en la glándula
fetal, deben ser estudiados postnatalmente con ecografía y suprarrenal izquierda. ¿Cuál de las siguientes afirmaciones
además con cistouretrografía miccional retrógrada. es cierta?
5) El tratamiento de elección consiste en realizar una pielo- 1) La edad inferior al año empeora el pronóstico.
plastia, aunque los resultados son mediocres y sólo el 50% 2) Si presentara metástasis hepáticas estaría contraindicado el
son satisfactorias. tratamiento quirúrgico.
3) Debe realizársele un aspirado de médula ósea como parte
71. Varón de 2 años en cuya historia clínica destaca haber del estudio de extensión.
padecido un proceso diarreico en los días previos. En la 4) El tumor que presenta es más frecuente en pacientes con
exploración destaca palidez, estupor y hematuria. Ha pre- hemihipertrofia.
sentado además una convulsión focal. Respecto al cuadro 5) El empleo de la gammagrafía con metayodobencilguanidina
que usted sospecha, señale cuál de las siguientes opciones ha sido abandonado por técnicas más específicas.
es FALSA:
1) Es la causa más frecuente de IRA en los niños pequeños. 76. Con respecto a los factores pronósticos del neuroblastoma,
2) Existe anemia hemolítica microangiopática y trombopenia señale la opción FALSA:
dentro del cuadro clínico. 1) El diagnóstico en menores de un año mejora el pronóstico.
3) La biopsia renal está indicada en casos de insuficiencia 2) La localización abdominal es la de peor pronóstico.
renal prolongada.
3) La amplificación del oncogén myc es un factor de mal pro-
4) El empleo de corticoides supone el tratamiento de elección.
nóstico.
5) La HTA ocurre en un tercio de pacientes y suele ser de difícil
control. 4) Las metástasis óseas son de peor pronóstico que las de
hígado, piel o médula ósea.
5) La aparición de un síndrome de opsoclono mioclono em-
72. En relación con el reflujo vesicoureteral en la infancia, señale peora el pronóstico.
la respuesta FALSA:
1) El reflujo vesicoureteral primario se debe a una anomalía 77. Niño de 3 años, que es remitido al hospital porque, en una
congénita de la unión ureterovesical. revisión rutinaria, su pediatra le ha palpado una masa abdo-
2) Si el reflujo es masivo puede llevar a IRC en la infancia y minal indolora en flanco derecho que no pasa línea media. La
requerir cirugía. madre refiere que al nacer “era muy gordo y tenía las tripas
3) En el reflujo de grado I y II el 80% de los casos se resuelven fuera”. Con respecto al tumor que Vd. sospecha, son ciertas
espontáneamente. todas las siguientes, EXCEPTO:
4) En el reflujo de grado IV y V la cirugía debe demorarse, bajo 1) En alrededor de un 10% de los pacientes puede aparecer
profilaxis antibiótica, hasta asegurarse plenamente que el hematuria macroscópica o microscópica.
reflujo no desaparece. 2) Para el estudio inicial, se recomienda la realización de una
5) Los resultados de la cirugía son excelentes. biopsia.

CTO Medicina • C/Francisco Silvela, 106 • 28002 Madrid • Tfno.: (0034) 91 782 43 30/33/34 • E-mail: secretaria@ctomedicina.com • www.grupocto.es
9
Pediatría

3)
Test 2V Distancia

Las metástasis más frecuentes son pulmonares.


Preguntas
Grupo CTO
CTO Medicina

82. Niña de 9 años, con febrícula y cansancio en los últimos 2


4) Se asocia a aniridia y hemihipertrofia. días. Hoy comienzan a aparecerle en los pies, las piernas y
5) El estadio V es el tumor bilateral. alguna por los muslos, máculas rosadas que van haciéndose
más oscuras, llegando incluso a palparse las que han aparecido
78. Señale la afirmación FALSA respecto a la PTI: antes, abultadas, aunque no son dolorosas ni pruriginosas.
1) Existe un importante descenso del número de plaquetas ¿Cuál es su diagnóstico de sospecha?
circulante, aunque el número de megacariocitos en médula 1) Varicela.
ósea es normal. 2) Enfermedad de Kawasaki.
2) Cursa con importante esplenomegalia. 3) Exantema súbito.
3) La complicación más grave de la enfermedad es la hemorragia 4) Púrpura de Schönlein-Henoch.
intracraneal. 5) Mononucleosis infecciosa.
4) En el tratamiento se emplea gammaglobulina i.v. y los cor-
ticoides. 83. Niña de 10 meses, que desde hace dos semanas presenta
5) El tiempo de hemorragia y el tiempo de retracción del un catarro de vías altas leve con febrícula. En los últimos
coágulo están alterados. días la tos ha ido en aumento, presentando en la actualidad
episodios de tos paroxística intensa, que acaban con un ruido
79. Niño de 2 años al que se ha realizado un análisis de sangre agudo inspiratorio y, a veces, van seguidos de vómitos. En la
por síndrome febril de 6 días de evolución, con buen estado exploración tiene 37,2 oC, buen estado general, y únicamente
general y sin foco infeccioso aparente. Los reactantes de fase se ausculta algún roncus aislado en ambos hemitórax. En el
aguda son normales, salvo ligera leucocitosis con predomi- hemograma destaca linfocitosis absoluta y la radiografía de
nio de linfocitos, por lo que usted tranquiliza a la madre tórax muestra infiltrados perihiliares bilaterales. ¿Cuál de
informándole de que lo más probable es que el niño tenga las siguientes afirmaciones es FALSA, con respecto a esta
una infección vírica y quizá el resultado de la serología dé patología?
el diagnóstico. Sin embargo, en el hemograma desataca una 1) En los lactantes de menos de 6 meses puede asociarse con
cifra de hemoglobina de 9,5 g/dl, con un volumen corpuscular alta mortalidad.
medio de 70 fL. Señale la respuesta INCORRECTA acerca de 2) Entre las posibles complicaciones destacan la apnea, OMA,
la patología que sospecha: neumonías y secuelas físicas secundarias a los esfuerzos de
1) Es la enfermedad hematológica más frecuente en la infancia. la tos.
2) Siempre hay que buscar pérdidas de sangre cuando nos 3) El tratamiento más adecuado es la administración de azi-
planteamos esta diagnóstico. tromicina e inmunoglobulinas específicas.
3) Es muy frecuente en los primeros 6 meses de vida. 4) Los adultos y adolescentes tosedores infectados son el prin-
4) El bajo peso al nacimiento es un factor de riesgo. cipal reservorio.
5) Una vez instaurado el tratamiento, el aumento de las cifras 5) Pueden persistir alteraciones transitorias de la función pul-
de hemglobina tarda 2-3 semanas en aparecer. monar en niños menores de 2 años.

80. Respecto a los tumores de la infancia, señale la afirmación 84. Varón de 6 años que, estando previamente bien, presenta fiebre
correcta: de 40 oC, de 24 horas de evolución, acompañada de vómitos,
1) La leucemia aguda es la patología oncológica más frecuente cefalea y dolor de garganta intenso. En la exploración encon-
de la infancia. tramos amígdalas grandes y enrojecidas, faringe hiperémica,
2) El astrocitoma se localiza principalmente en fosa anterior. lengua de color rojo intenso, petequias en velo del paladar y
3) El tumor sólido extracraneal más frecuente es el nefroblas- úvula, exantema rojo intenso, maculopapuloso, puntiforme,
toma. de predominio en pliegues, así como enrojecimiento difuso
4) No se debe biopsiar un neuroblastoma por el riesgo de en la cara, que respeta triángulo perinasobucal. Señale la
sangrado. afirmación correcta:
5) Los niños con tumor de Wilms tienen un aspecto de muy 1) La contagiosidad remite nada más instaurar la antibioterapia.
enfermos. 2) El exantema afecta siempre a palmas y plantas.
3) Puede aparecer como complicación la afectación renal,
81. Con respecto al diagnóstico del niño VIH positivo, señale cuál pero no la fiebre reumática.
de las siguientes afirmaciones es FALSA: 4) Las pruebas de detección rápida de antígenos de S. pyogenes
1) En lactantes mayores de 18 meses, el diagnóstico se hace y el cultivo del frotis faríngeo son las pruebas de laboratorio
igual que en adultos. más útiles para establecer el diagnóstico.
2) La edad de presentación, el recuento de linfocitos CD4/CD8 5) El tratamiento de elección es la penicilina benzatina durante
y la carga viral son factores que condicionan el pronóstico. 3 días.
3) En un niño menor de 18 meses puede hacerse el diagnós-
tico de infección por VIH, si existen dos determinaciones 85. Señale cuál de las siguientes afirmaciones es FALSA, respecto
positivas al realizar el cultivo viral, la PCR o la determinación al sarampión:
del Ag p24. 1) El período de máxima contagiosidad abarca la fase prodró-
4) Al nacimiento, todos los hijos de madre VIH+ tienen anti- mica de la enfermedad.
cuerpos (Acs) tipo Ig A frente al VIH. 2) Son raros los casos de sarampión en lactantes menores de
5) Para diagnosticar la serorreversión de los Ac en un niño entre 4-6 meses.
los 6 y 18 meses, son necesarios dos test de ELISA negativos. 3) Las manchas de Koplik son patognomónicas.

CTO Medicina • C/Francisco Silvela, 106 • 28002 Madrid • Tfno.: (0034) 91 782 43 30/33/34 • E-mail: secretaria@ctomedicina.com • www.grupocto.es
10
4)
Preguntas
Grupo CTO
CTO Medicina

Coincidiendo con la aparición del exantema, suele producirse


Test 2V
Distancia Pediatría

90. Paciente de 17 meses, que acude a Urgencias por convulsión


una brusca elevación de la temperatura. tónico-clónica generalizada y fiebre de 39 oC. En la analítica
5) El exantema se inicia en el tronco, y desde ahí se extiende a destaca leucocitosis y neutrofilia. Se ingresa para observación.
la cara y extremidades, desapareciendo en el mismo orden. A los 3 días desaparece la fiebre, apareciendo un exantema.
Señale la opción verdadera, con respecto al proceso que
86. Una de las siguientes afirmaciones NO es correcta, respecto padece este paciente:
a las complicaciones del sarampión: 1) Si al tercer día de evolución desaparece la fiebre y aparece
1) Una de las complicaciones más frecuentes es la otitis media. un exantema vesiculoso en tronco y miembros inferiores,
2) La neumonía de células gigantes de Hecht es más frecuente podemos establecer el diagnóstico de roséola.
que la neumonía causada por sobreinfección bacteriana. 2) La lengua en fresa roja es un hallazgo frecuente en esta
3) Hasta el 50% de los niños con sarampión no complicado patología.
tienen alteraciones en el EEG, sin otros signos de alteración 3) El agente etiológico más frecuente es el herpesvirus humano
del SNC. tipo 6.
4) No existe correlación entre la gravedad del sarampión y la 4) Es muy frecuente la alteración del LCR.
de la encefalomielitis aguda. 5) El tratamiento de elección es penicilina v.o.
5) Puede aparecer reactivación de una tuberculosis preexistente,
acompañando al sarampión. 91. Una niña de 3 años, previamente sana, bien vacunada, co-
menzó hace 3 días con unas lesiones cutáneas pequeñas,
87. Niña de 2 años, que presenta desde ayer catarro de vías altas y maculosas y rojas, que evolucionaron a vesículas claras en
fiebre de 38 oC. Hoy acude por aparecer un exantema maculo- tórax, cara, muslos y brazos. En la exploración destacan,
papuloso rosado, no confluente, no puntiforme ni pruriginoso, además, algunas costras y pústulas, tres pequeñas úlceras
en cara, tronco y extremidades. En la exploración, además en la lengua y paladar, y dos lesiones similares en los labios
del exantema, destacan conjuntivas hiperémicas, múltiples menores. ¿Cuál es su diagnóstico?
adenopatías, dolorosas a la palpación, en región cervical pos- 1) Primoinfección herpética.
terior y postoccipital, así como un polo de bazo. La faringe está 2) Síndrome pie-mano-boca.
levemente hiperémica y se observan pequeñas manchas rojas 3) Erupción variceliforme de Kaposi.
en el velo del paladar. El diagnóstico más probable es: 4) Varicela.
1) Sarampión. 5) Herpangina.
2) Rubéola.
3) Escarlatina. 92. Un niño de 13 años presenta dolor y tumoración en la
4) Mononucleosis infecciosa. mandíbula derecha, que borra ángulo y ha ido aumentando
5) Exantema súbito. rápidamente en pocas horas. Además, presenta fiebre de
38,5 oC y cefalea. Señale la afirmación cierta:
88. Niño de 14 meses traído a Urgencias por su madre al observar 1) El 95% de las infecciones por este paramixovirus producen
desconexión del medio, rigidez tónica, versión ocular y cianosis clínica.
perioral, de tres minutos de duración, con somnolencia pos- 2) En más del 65% de los enfermos se detecta pleocitosis en
terior durante cinco minutos. Al llegar a Urgencias se observa el LCR, aunque no presenten manifestaciones clínicas de
temperatura de 39 oC. La exploración neurológica es normal y la meningoencefalitis.
otoscopia muestra signos de otitis media serosa. La madre refiere 3) Con frecuencia se acompaña de pancreatitis clínica.
que a la edad de seis meses tuvo un episodio similar, también 4) La tumefacción parotídea es indolora y existen algunos signos
coincidiendo con fiebre. ¿Cuál sería la actitud más correcta? de inflamación de la piel suprayacente.
1) Observación durante 24 horas e iniciar tratamiento con 5) En un elevado porcentaje de los varones aparece esterilidad
carbamacepina. tras la orquitis postparotiditis.
2) Realizar TC craneal y punción lumbar.
3) TC craneal, punción lumbar y hemocultivos seriados. 93. Varón de 13 años que, desde hace una semana, presenta un
4) EEG e iniciar tratamiento con ácido valproico. cuadro de malestar general, fatiga y dolor abdominal. Desde
5) Observación, tratamiento antitérmico y seguimiento ambu- hace 24 horas presenta además odinofagia y aparición de
latorio. varias adenopatías cervicales posteriores. En la exploración
destaca temperatura de 39 oC, varias linfadenopatías de 1,5
89. Señala cuál de las siguientes afirmaciones, respecto a la cm de diámetro en región cervical posterior, así como polo
rubéola y sus complicaciones, es FALSA: de bazo palpable, a 3 cm del reborde costal izquierdo, e
1) La artritis suele afectar a mujeres, grandes articulaciones y hipertrofia amigdalar con exudado blancoamarillento. Con
suele resolverse sin dejar secuelas. este cuadro, usted ya se ha hecho su impresión diagnóstica,
2) Pueden aparecer complicaciones neurológicas, siendo éstas y sabe que el proceso en cuestión se caracteriza por todo lo
mucho menos frecuentes que en el caso del sarampión. siguiente, EXCEPTO:
3) La profilaxis con gammaglobulina está indicada en mujeres 1) Su agente etiológico más probable es el virus de Epstein-
embarazadas durante el primer trimestre de la gestación. Barr.
4) En casos de rubéola congénita, el paciente puede excretar 2) Se trata de un virus linfotropo, que afecta a los linfocitos de
virus durante meses. estirpe T.
5) La relación entre formas subclínicas/enfermedad patente es 3) Aunque la hepatomegalia sea poco frecuente, sabe que hasta
de 2/1. en un 50% es posible detectar elevación de las transaminasas.

CTO Medicina • C/Francisco Silvela, 106 • 28002 Madrid • Tfno.: (0034) 91 782 43 30/33/34 • E-mail: secretaria@ctomedicina.com • www.grupocto.es
11
Pediatría

4)
Test 2V Distancia

Dentro de la evolución de la enfermedad, hasta en un 80%


Preguntas

2)
Grupo CTO
CTO Medicina

Fenobarbital.
puede aparecer un exantema maculopapuloso al administrar 3) ACTH.
ampicilina. 4) Paraldehído.
5) Para el diagnóstico, es útil la determinación de Ac heterófilos. 5) Etosuximida.

94. ¿Cuál de las siguientes es una característica propia del SIDA 100. Acude a nuestra consulta un niño de 8 años, hiperactivo sin
infantil respecto al SIDA en los adultos? una finalidad, tanto en casa como en sus actividades escola-
1) Mayor tiempo de incubación. res, que además es impulsivo, agresivo con sus compañeros,
2) Hipergammaglobulinemia tardía. tiene crisis de frustración con facilidad y dificultades en su
3) Menor número de infecciones bacterianas. rendimiento escolar. ¿Cuál de las siguientes respuestas consi-
4) Mayor linfopenia. dera FALSA, respecto al cuadro que parece presentar nuestro
5) Menor número de infecciones oportunistas. paciente?
1) Se considera que puede existir una predisposición genética.
95. Respecto al síndrome de muerte súbita infantil o del lactante 2) Se ha relacionado con alteración en receptores dopami-
(SMSL), señale cuál de los siguientes NO constituye un factor de nérgicos.
riesgo: 3) Es un trastorno más frecuente en varones.
1) Antecedentes familiares. 4) Se han demostrado alteraciones en estudios de neuroimagen
2) Prematuridad. y de metabolismo cerebral.
3) Posición para dormir en decúbito supino. 5) El tratamiento se basa en fármacos sedantes y apoyo psico-
4) El sexo masculino. pedagógico.
5) El tabaquismo materno.

96. En la tetralogía de Fallot NO es cierto que:


1) La clínica depende del grado de obstrucción a la salida del
ventrículo derecho.
2) Aparece cianosis progresiva y disnea de esfuerzo, que mejora
en posición de cuclillas.
3) En la Rx de tórax aparece una cardiomegalia con pedículo
cardíaco estrecho y plétora pulmonar.
4) En la auscultación cardíaca se detecta un soplo sistólico en
borde esternal izquierdo.
5) En el ECG aparece hipertrofia de ventrículo derecho.

97. Señale cuál de los siguientes signos radiológicos es típico de


la coartación aórtica:
1) Silueta cardíaca “en zueco”.
2) Muescas o escotaduras costales.
3) Signo “del hombre de nieve”.
4) Silueta cardíaca estrecha u ovalada.
5) Síndrome “de la cimitarra”.

98. A la guardia llega un lactante de 15 meses con una convulsión


tónico-clónica generalizada. Sobre la actitud que Vd. adoptaría,
señale la respuesta INCORRECTA:
1) Lo más frecuente es que se trate de una convulsión febril,
por lo que se debe establecer la temperatura del paciente.
2) Lo habitual es que la duración sea inferior a 10 minutos.
3) Si el inicio es focal, se debe realizar un EEG.
4) Hasta un 95% de los casos presentan recurrencias posteriores,
que suelen ceder hacia los 5 años.
5) El tratamiento profiláctico con diacepam rectal sólo se
instaura ante convulsiones febriles muy frecuentes.

99. Varón de 6 meses, con movimientos de flexión brusca de cuello,


brazos y piernas hacia el tronco, con posterior extensión de
miembros, que aparecen al despertar. Entre sus antecedentes,
no destaca ningún factor predisponente. La exploración neu-
rológica y la TC craneal son normales y en el EEG se observa
un patrón de hipsarritmia. Usted sabe que el tratamiento de
primera elección, de entre los siguientes, es:
1) Valproato sódico.

CTO Medicina • C/Francisco Silvela, 106 • 28002 Madrid • Tfno.: (0034) 91 782 43 30/33/34 • E-mail: secretaria@ctomedicina.com • www.grupocto.es
12
Distancia

Test 2VPediatría
Grupo CTO
CTO Medicina

Comentarios

Pregunta 1.- R: 4 • El meconio se elimina en las primeras 48 horas de vida, y la


• Los niños duplican su peso a los 5 meses de vida, lo triplican primera micción suele ocurrir en las primeras 24 horas.
al año y lo cuadruplican a los dos años. Duplican la talla del • En el RN, tanto la capacidad de filtración como la de concen-
nacimiento con 4 años. tración de la orina están mermadas.
• El cierre de la fontanela anterior ocurre entre los 9 y los 18 meses • La frecuencia cardíaca normal en un RN oscila entre 120-160
de vida, y la posterior se cierra antes (alrededor de los 2 meses). lpm (se consideraría taquicardia en un adulto) y la FR, es de
• El perímetro torácico en el momento del nacimiento es menor unas 30-40 rpm (se consideraría taquipnea en un adulto).
que el del perímetro cefálico (éste último mide 35 cm aproxi- • Los RN presentan leucocitosis fisiológica (>11.000 leucocitos/
madamente), y ambos se igualan alrededor del año de vida. mm3), lo patológico es la leucopenia y nacen con una Hb alta
Posteriormente el perímetro torácico es mayor. (16-18 g/dl).
• La piel del pretérmino es fina, friable, hace equimosis con
facilidad, y presenta lanugo (vello fino y evanescente). El vér- Pregunta 3.- R: 3
nix caseoso, unto sebáceo de la piel, está presente en los RN Se trata de un caso típico de hernia diafragmática congénita. La
a término. Los RN postérmino tienen una piel quebradiza y posterior o de Bochdaleck es el tipo de hernia diafragmática más
apergaminada, hecho que les confiere un aspecto “de viejillo”. frecuente, sobre todo izquierda. Consiste en un orificio grande en
Los quistes de millium, el eritema tóxico, la melanosis pustulosa la parte posterior izquierda del diafragma a través del cual vísceras
y los angiomas planos son lesiones sin significado patológico. abdominales migran hacia el tórax, con lo que el abdomen aparece
• Recuerda los trastornos relacionados con la fontanela anterior excavado (vacío de contenido) y hay un compromiso respiratorio se-
grande/retraso de cierre. La prueba clave para su valoración: vero secundario a la hipoplasia y compresión pulmonar. Es un cuadro
ecografía transfontanelar. severo que aboca a la instauración de hipertensión pulmonar que ya
está presente en el nacimiento.
• Acondroplasia
• Disostosis cleidocraneal El diagnóstico en la mayoría de los casos se hace por ecografía
• Síndrome de Down prenatal. Cuando no es así, se realiza Rx de tórax al nacimiento
• Hipotiroidismo congénito donde advertiremos la presencia de asas intestinales en tórax, y el
• Hidrocefalia mediastino estará desplazado por el efecto masa de éstas. Si tras
• Aumento presión intracraneal la radiografía hay dudas con malformaciones pulmonares quísticas,
• Raquitismo por deficiencia de vitamina D hay que realizar una ecografía. El cierre quirúrgico se hará a las
• Menos frecuentes: 24-72 horas tras estabilizar situación hemodinámica del paciente,
-CIR primero durante 10-12 horas forzaremos una situación de alcalosis
-Otras anomalías cromosómicas (el pH alcalino vasodilata las arterias pulmonares) para minimizar
-Infección congénita (sífilis, rubéola) la hipertensión pulmonar.
-Síndromes dismórficos (VATER, Apert)
Pregunta 1. Trastornos relacionados con la fontanela La hernia diafragmática congénita es más frecuente en algunos
anterior grande. síndromes congénitos, como las trisomías 21, 13 y 18. En un 20-30%
de los casos hay otras alteraciones asociadas como malrotación in-
Pregunta 2.- R: 5 testinal (la más frecuente), cardiopatías y malformaciones vasculares.
• En un RN normal, al poco de nacer se cierran los cortocircuitos
de la circulación fetal: ductus arterioso, conducto venoso de Pregunta 4.- R: 4
Arancio y foramen oval. No olvides repasar el test de Apgar antes del examen MIR.

CTO Medicina • C/Francisco Silvela, 106 • 28002 Madrid • Tfno.: (0034) 91 782 43 30/33/34 • E-mail: secretaria@ctomedicina.com • www.grupocto.es
1
Pediatría
Test 2V Distancia
0
Comentarios
Grupo CTO
CTO Medicina

1 2

Esfuerzo
Ausente Lento, irregular Bueno, llanto
respiratorio

Respuesta
a la introducción Sin respuesta Mueca Tos o estornudo
de una sonda nasogástrica

Frecuencia cardíaca Ausente Menos de 100 Más de 100

Tono Ligera flexión


Débil Movimientos activos
muscular de extremidades

Cuerpo sonrosado,
Color Azul, pálido Totalmente sonrosado
extremidades cianóticas

Pregunta 4. Test de Apgar.

Pregunta 5.- R: 3 feriores (C7 y C8) del plexo braquial (respuesta 1 correcta). El recién
El test de Apgar se debe realizar a todo recién nacido, indepen- nacido presenta la mano caída con el reflejo de prensión palmar del
dientemente de su edad gestacional, al minuto y a los 5 minutos y lado afecto ausente. Sin embargo, el reflejo de Moro estará presente
posteriormente sólo se repetirá cada 5 minutos si la puntuación a los ya que las raíces superiores no se han afectado (respuesta 2 falsa).
5 minutos es inferior de 7 (respuesta 3 correcta). Si se lesiona también la primera raíz torácica aparecerá síndrome de
Horner asociado (respuesta 4 correcta).
Entre los parámetros que valora se encuentran: color, frecuencia
cardíaca, respuesta a la estimulación, tono muscular y esfuerzo res- La parálisis de Erb-Duchenne es más frecuente (respuesta 5 correc-
piratorio (NO frecuencia respiratoria). ta). Mientras que la parálisis de Klumpke es favorecida por el parto
de nalgas (respuesta 3 correcta), la de Erb se produce por distocia de
A pesar de una puntuación baja en los primeros minutos de vida, hombros en presentaciones cefálicas.
no sirve para establecer valoraciones pronósticas. Una puntuación
inferior a 3, mantenida más allá de los 20 minutos de vida, sí puede
predecir una elevada morbimortalidad. PARÁLISIS BRAQUIAL P. ERB-DUCHENNE P. KLUMPKE

La palidez cutánea generalizada supone una puntuación de 0 en el Raíces (C4) - C5 - C6 C7 - C8 - (T1)


test de Apgar (ver el cuadro del comentario de la pregunta anterior). Brazo en adducción
Clínica Mano caída
y rotación interna
Pregunta 6.- R: 3 No presente o asimé-
R. Moro Presente
Debes conocer el cefalohematoma y sus diferencias con respecto trico
al caput succedaneum.
R. prensión palmar Presente No presente

Asociaciones C4 - parálisis frénica T1 - s. Horner

CAPUT SUCCEDANEUM CEFALOLOHEMATOMA Pregunta 7. Parálisis braquial.


Edema de tejido celular
Hemorragia subperióstica
subcutáneo
Inicio En el momento del parto Horas después del parto Pregunta 8.- R: 3
Respeta El enunciado de esta pregunta muestra:
No Sí
suturas • Un RN macrosómico.
Resolución En unos días 2 semanas-3 meses • Nacido de nalgas.
• En el contexto de un parto laborioso y traumático (fractura de
Piel
suprayacente
Equimótica a veces Normal clavícula, parálisis braquial inferior, caput grande).
• Y que muestra signos clínicos de shock (polipnea, hipotensión).
Pregunta 6. Diferencias entre caput succedaneum y cefalohematoma.

Con estas características (parto traumático, presentación


podálica, macrosomía, shock), rápidamente has de pensar en
Recuerda que la incisión y aspiración del cefalohematoma está la posibilidad de una hemorragia suprarrenal. A su vez ésta se
contraindicada por el riesgo de sobreinfección y que rara vez puede expresa como:
producir, si es de suficiente cuantía, anemia e ictericia llegando incluso • Masa palpable en uno de los flancos, en la exploración (opción 1).
a requerir transfusión y/o fototerapia. • Anemización, como cualquier hemorragia (opción 2), con aumento
de la oferta de bilirrubina indirecta e ictericia (opción 4).
Pregunta 7.- R: 2 • En el plano analítico, cabe reseñar la aparición de hipoglucemia
Caso clínico de una parálisis braquial inferior, también conocida mantenida (lo más típico), y una situación de hipoaldosteronis-
como parálisis de Klumpke. Se debe a la afectación de las raíces in- mo, con sodio bajo y potasio alto (opción 5).

CTO Medicina • C/Francisco Silvela, 106 • 28002 Madrid • Tfno.: (0034) 91 782 43 30/33/34 • E-mail: secretaria@ctomedicina.com • www.grupocto.es
2
Comentarios
Grupo CTO
CTO Medicina

La opción 3 es falsa. No tiene por qué haber desviación izquierda


Test 2V Distancia Pediatría

Deben administrarse antibióticos (ampicilina + gentamicina)


en el hemograma. Este hecho sería típico de la sepsis neonatal. hasta resultados de los cultivos, ya que una sepsis con partici-
pación pulmonar puede dar el mismo patrón radiológico.
Pregunta 9.- R: 5
Una tumoración abdominal, fácilmente reductible, en la base del
cordón umbilical y cubierta de piel, coincide con la descripción de
una hernia umbilical (respuesta 5 correcta). Si la tumoración estuviera
recubierta por peritoneo se trataría de un onfalocele y, si no tiene
cubierta externa, sería una gastrosquisis.

El granuloma se trata de un tejido blando, granular y de color


rojizo o rosado, que puede tener una secreción mucopurulenta. La
persistencia de uraco se manifestaría como una fístula que exuda un
líquido ácido.

Pregunta 10.- R: 3
El enunciado de esta pregunta muestra:
• A un RN casi a término (36 semanas).
• Con signos de dificultad respiratoria (retracción subesternal,
subcostal y quejido) y polipnea.
• De aparición en las primeras dos horas de vida.
• Que evoluciona favorablemente con la administración de oxígeno.

Recuerda este “retrato robot” para el MIR, pues corresponde a


un distrés respiratorio tipo II o taquipnea transitoria del RN o
síndrome de Avery (respuesta 2 correcta). Esta variante de dificultad Pregunta 11. Factores que contribuyen en la patogenia
respiratoria presenta: de la EMH. "Círculo vicioso".
• Como desencadenante, un parto por cesárea o un parto vaginal
rápido (respuesta 1 correcta), con la incapacidad secundaria Pregunta 12.- R: 4
para llevar a cabo una reabsorción correcta del líquido que Una de las complicaciones de la enfermedad de membrana hialina es
anega los pulmones intraútero. la aparición de un ductus arterioso persistente. El “retrato robot” de
• Un desenlace rápido, o mejoría sintomática en los primeros 1-2 DAP puede quedar delimitado con lo que aparece en esta pregunta:
días de vida (respuesta 3 falsa). • Pretérmino.
• El patrón radiológico clásico consiste en la aparición de pulmo- • Con enfermedad de membrana hialina.
nes levemente congestivos, líquido en cisuras, derrame pleural • Que, después de la fase de mejoría sintomática de la misma (3-5
de pequeño tamaño (respuesta 5 correcta). días de vida), se deteriora bruscamente, con nueva aparición
• Recuerda que el tratamiento se basa en oxigenoterapia hasta la de distrés respiratorio.
resolución del cuadro, generalmente en pequeñas cantidades. • Retención de CO2 en el plano analítico.
A veces se benefician de CPAP. • Aparición de soplo continuo “en maquinaria” o de Gibson y
pulsos exaltados, muy llamativos.
Pregunta 11.- R: 3 • En la Rx de tórax aparece cardiomegalia e hiperaflujo pulmonar.
La enfermedad de membrana hialina se caracteriza por: El diagnóstico es con ecocardiograma.
• Dificultad respiratoria que afecta a un RN pretérmino, que pro- • En caso de deterioro progresivo, está indicado cierre farmacoló-
gresa en las primeras horas de vida, y que se muestra refractaria gico con indometacina. Como alternativa ibuprofeno o cirugía.
a la aplicación de oxígeno indirecto (respuesta 1 correcta).
• Se debe a un déficit de surfactante y a una inmadurez pulmonar, Al no ser una cardiopatía cianosante, las opciones 1 y 3 quedan
con cociente L/E<2 y ausencia de fosfatidilglicerol (respuesta 2 descartadas. La coartación de aorta cursa con pulsos débiles. La CIV
correcta). cursa sin cianosis, pero el soplo aparece desde las primeras horas de
• A nivel analítico destaca una insuficiencia respiratoria mixta, vida, y es pansistólico (no continuo).
con hipoxia franca (respuesta 3 falsa), llamativa, importante.
• A nivel radiológico, los hallazgos clásicos son: colapso pulmonar, Pregunta 13.- R: 4
patrón reticulogranular con broncograma aéreo de predominio El enunciado de la pregunta describe a la perfección el síndrome
basal, rico en atelectasias, con aspecto global de vidrio esme- de aspiración meconial (SAM):
rilado. Su grado extremo es el “pulmón blanco”, totalmente • Un RN postérmino (44 semanas de edad gestacional).
atelectásico (respuesta 4 correcta). • Con antecedentes de sufrimiento fetal (recuerda la secuencia
• El tratamiento consiste en soporte respiratorio pero no se debe SFA... hipoxia... contracción intestinal con relajación del esfínter
reanimar con oxígeno al 100% (la hiperoxia produce disminu- anal... eliminación de meconio).
ción de la producción de surfactante, retinopatía, reducción • Que presenta distrés respiratorio de inicio precoz.
del flujo cerebral y riesgo de displasia broncopulmonar); y en • Con hiperinsuflación torácica (el meconio provoca atropamiento
la administración endotraqueal de surfactante (mejora la super- aéreo por un mecanismo valvular, que permite la entrada de
vivencia pero no la incidencia de displasia broncopulmonar). aire pero limita su salida), con tendencia a un rebosamiento

CTO Medicina • C/Francisco Silvela, 106 • 28002 Madrid • Tfno.: (0034) 91 782 43 30/33/34 • E-mail: secretaria@ctomedicina.com • www.grupocto.es
3
Pediatría
Test 2V Distancia

del alvéolo, con posibilidad de ruptura del mismo, y aparición


Comentarios
Grupo CTO
CTO Medicina

de líquido los alveolos del pulmón. La clínica se inicia en las prime-


de aire ectópico en sus distintas modalidades (en el enunciado ras horas de vida, que cede y mejora con medidas poco agresivas
aparece un neumotórax: hiperresonancia pulmonar, abolición (oxígeno en incubadora). Suelen alcanzar la resolución clínica en un
del murmullo vesicular, dolor con inquietud, apnea). plazo máximo de 3 días.
• Recuerda la radiología típica: infiltrados algodonosos, pulmones
hiperinsuflados, áreas ampollosas y aparición de aire ectópico. Lo que establece el diagnóstico, aparte de la evolución clínica, es la
• El tratamiento se basa en medidas generales y ventilación Rx de tórax, donde se aprecia aumento de marcas vasculares, líquido
asistida, si es necesario. en las cisuras y discreta hiperinsuflación que hace que el diafragma
se aplane. A diferencia del resto de opciones, el distress respiratorio
Pregunta 14.- R: 2 en la taquipnea transitoria del RN suele ser leve-moderado, con poca
Caso clínico de una displasia broncopulmonar, que se define como o ninguna alteración gasométrica.
la necesidad de oxígeno a los 28 días de vida, típico de neonatos
que tienen como antecedente haber padecido una enfermedad de Pregunta 16.- R: 5
membrana hialina. Si no hay antecedentes de hialina, llamaremos Uno de cada doscientos RN alimentados al pecho presenta un
a la demanda de oxígeno síndrome de Wilson-Mikity (respuesta 4 cuadro que se conoce como síndrome de Arias, y que consiste en
correcta). La DBP provoca una elevación de las resistencias pulmo- la aparición de ictericia. Ésta se suele extender entre los días 7 y 30
nares, por tanto, fomenta a aparición de una hipertensión pulmonar de vida. Se debe a que en la leche materna existen unas sustancias
secundaria que sobrecarga el ventrículo derecho de forma crónica, (pregnanodiol, ácidos grasos de cadena larga) que inhiben la glucuronil
pudiendo llegar a la ICC (respuesta 1 correcta). transferasa fetal, dando como resultado un aumento de la bilirrubina
a expensas de fracción indirecta.
El diagnóstico se establece por la historia clínica y la Rx de tórax,
donde es típico encontrar el llamado “patrón de esponja” que con- Generalmente, no reviste trascendencia y no se han descrito casos
siste en áreas más claras que alternan con otras de mayor densidad de kernicterus. Por tanto, la actitud del pediatra ha de ser tranquili-
(respuesta 3 correcta). zadora. No está indicado retirar la lactancia materna.

El tratamiento se basa en oxigenoterapia, restricción de líquidos y Pregunta 17.- R: 2


diuréticos (respuesta 2 falsa), para minimizar la sobrecarga pulmonar El RNPT, sobre todo si sufre patología importante, es especialmente
y cardiaca, broncodilatadores y corticoides (sólo en casos graves). La vulnerable a la aparición de toxicidad neurológica por bilirrubina in-
mayoría de los niños suelen tener un curso favorable, alcanzando la directa (kernicterus). Es conveniente memorizar las cifras por encima
normalidad de la función pulmonar hacia los 2 años de vida (respuesta de las cuales hay que exanguinotransfundir:
5 correcta). Hay un pequeño porcentaje que evolucionan hacia un • En RNPT: por encima de 17 mg/dl de bilirrubina.
cuadro de hipertensión pulmonar persistente. Las dos causas más • En RNT, por encima de 25 mg/dl.
frecuentes de muerte en estos niños son la ICC derecha y la bron-
quiolitis necrotizante. Nuestro neonato presenta 20 mg/dl de bilirrubina total e importante
patología de base; en situaciones como esta debemos ser agresivos, y
Pregunta 15.- R: 4 proceder a la exanguinotransfusión (respuesta 2 correcta).
El cuadro clínico se refiere a una taquipnea transitoria del RN, Sd.
de Avery, pulmón húmedo o SDR tipo II. La presentación típica es Pregunta 18.- R: 2
un RN nacido por cesárea o por parto vaginal rápido, lo que supone Una ictericia precoz, de aparición en las primeras 24-48 horas de
que el tórax no ha estado sometido a la presión positiva que exprime vida, ha de obligarnos a pensar en tres grandes patologías:

Tipo de Ictericia Concentración


DIAGNÓSTICO Observaciones
BR APARECE DESAPARECE máxima de BR

Ictericia fisiológica
Indirecta 2-3 días 4-5 días 2-3 días Relación con el
RNT
Indirecta 3-4 días 7-9 días 6-8 días grado de madurez
RNPT

Hipoxia, dificultad respiratoria,


HiperBR 2ª a factores déficit carbohidratos.
metabólicos Indirecta 2-3 días Variable 1ª semana Influencias hormonales:
RNT Indirecta 3-4 días Variable 1ª semana cretinismo, hormonas.
RNPT Fact. Genéticos: Crigler-Najjar, hiperBR
familiar transitoria.

Cuadros hemolíticos y Eritroblastosis, hemólisis congénita,


Indirecta Primeras 24 horas Variable Variable
hematomas picnocitosis infantil, vit. K, hematomas.

Sepsis bacteriana, pielonefritis,


Factores hemolíticos y Indirecta y
Primeras 24 horas Variable Variable hepatitis, toxoplasmosis,
hepatotóxicos mixtos directa
enf. inclusiones citomegálicas, rubéola

Indirecta y Atresia biliar, galactosemia, hepatitis e


Lesión hepatocelular 2-3 días Variable Variable
directa infecciones.

Pregunta 18. Rasgos diagnósticos para los diferentes tipos de ictericia neonatal.

CTO Medicina • C/Francisco Silvela, 106 • 28002 Madrid • Tfno.: (0034) 91 782 43 30/33/34 • E-mail: secretaria@ctomedicina.com • www.grupocto.es
4
Comentarios
Grupo CTO
CTO Medicina

• Hemorragia aguda en el contexto de un parto traumático.


Test 2V Distancia Pediatría

contrario taquipnea, llanto, hipotonía, letargia, rechazo del alimento


• Isoinmunización (test de Coombs directo positivo, anemización). y movimientos oculares anómalos. La incidencia disminuye con la
• Infecciones (sepsis neonatal, infecciones connatales). alimentación precoz.

Hay otras ictericias de aparición más tardía (> 1 mes): hipotiroi- Las crisis convulsivas no son causa de hipoglucemia, por el contrario
dismo (respuesta 2 falsa), galactosemia, ictericia obstructiva, Gilbert, son consecuencia de la misma, manifestación clínica de hipoglucemia
metabolopatías, etc. grave (respuesta 2 falsa). El resto de respuestas propuestas son ejemplos
de los cuatro grupos fisiopatológicos por los que se puede dar esta situa-
Pregunta 19.- R: 5 ción: hiperinsulinismo (hijo de madre diabética, eritroblastosis fetal grave,
Estamos ante un RN con un cuadro de enterocolitis necrotizante hiperinsulinismos familiares), disminución de las reservas hepáticas de glu-
(NEC). La presentación típica es en un RNPT que a la segunda semana cógeno por mala nutrición intrauterina (CIR), aumento desproporcionado
de vida inicia cuadro de vómitos, distensión abdominal y deposiciones de las necesidades metabólicas (enfermedades graves como cuadros de
con sangre. dificultad respiratoria importante, insuficiencia cardíaca) y enfermedades
metabólicas congénitas (glucogenosis, galactosemia, alteraciones del ciclo
Si el niño no tiene signos de perforación intestinal, el tratamiento de la urea, alteraciones de la oxidación de los ácidos grasos).
será dieta absoluta y antibioterapia que cubra anaerobios y gramnega-
tivos. El tratamiento será quirúrgico en caso de perforación intestinal Pregunta 22.- R: 1
(visible si neumoperitoneo) o sepsis refractaria al tratamiento médico Los hijos de madre consumidora de opiáceos no tienen más
(respuesta 5 correcta). riesgo de desarrollar malformaciones congénitas (respuesta 1 falsa).
Recuerda que, por el contrario, la cocaína no suele producir síndro-
Pregunta 20.- R: 3 me de abstinencia pero sí malformaciones congénitas (microcefalia,
La enterocolitis necrotizante (NEC) es una lesión isquémico- anomalías digestivas, renales), muerte súbita, alteraciones neurológicas
necrótica que suele afectar a íleon distal y colon proximal, y que se y de conducta. El alcohol tampoco produce síndrome de abstinencia.
sigue habitualmente de sepsis bacteriana a partir de foco digestivo.
El síndrome de abstinencia a heroína aparece más precozmente
Se consideran causas que predisponen a la isquemia: la polici- que el de metadona (respuesta 2 correcta), es menos convulsivógeno
temia, el inicio de la alimentación muy pronto y con elevados volú- (respuesta 3 correcta), y desaparece antes.
menes y concentraciones y situaciones de hipoxia y de bajo gasto.
También se han involucrado a diversos patógenos como E. coli, C. Los opiáceos protegen frente a la EMH (respuesta 5 correcta),
perfringens, Rotavirus y S. epidermidis, pero en la mayoría de los casos y su abstinencia, en el período neonatal se trata con morfina oral
no se detecta ningún germen responsable. La lactancia materna es (respuesta 4 correcta).
un factor protector.
Pregunta 23.- R: 3
Cursa con distensión abdominal (que sería el primer signo) y de- Problemas observados frecuentemente en HMD:
posiciones sanguinolentas en la 2ª semana de vida. Con frecuencia • Mayor mortalidad fetal y neonatal.
es un cuadro de inicio insidioso, que acaba dando lugar a una sepsis, • Polihidramnios.
pudiendo acabar en shock y muerte. • Macrosomía con visceromegalia, si la madre no tiene vasculo-
En la Rx simple de abdomen encontraremos: patía; si la tiene, CIR.
• Neumatosis intestinal (signo diagnóstico de la NEC) • Estenosis subaórtica, con hipertrofia septal asimétrica.
• Edema de asas. • EMH, por disminución de la síntesis de surfactante.
• Patrón “en miga de pan”. • Alteraciones metabólicas: hipoglucemia, con frecuencia asin-
• Asa fija. tomática (respuesta 3 incorrecta).
• Gas en la vena porta. • Policitemia y sus consecuencias.
• Neumoperitoneo;si existe perforación. • Mayor incidencia de malformaciones congénitas:
- Cardíacas (malformaciones más frecuentes en el HMD).
Manejo: - Colon izquierdo hipoplásico (malformación digestiva más
• Dieta absoluta, fluidoterapia y descompresión nasogástrica. frecuente).
• Antibióticos que cubran gérmenes anaerobios y gramnegativos. - Agenesia lumbosacra (malformación más característica).
• Tratamiento quirúrgico en caso de perforación intestinal o sepsis
refractaria al tratamiento médico. Pregunta 24.- R: 3
Los factores de riesgo para padecer una sepsis por C. Albicans son:
En este caso, la presencia de gas en la pared intestinal (neumatosis presencia de catéteres, nutrición parenteral prolongada, tratamiento
intestinal) no es indicación de cirugía urgente (respuesta incorrecta con antibióticos, hospitalización prolongada, colonización cutánea por
3). Sí lo sería la presencia de neumoperitoneo. Candida. No es más frecuente en el hijo de madre diabética (respuesta
1 falsa). Recuerda que Candida es más frecuente en diabéticos de
Pregunta 21.- R: 2 todas las edades, pero no por ello en sus hijos.
Recuerda que a menudo la hipoglucemia es asintomática, espe-
cialmente en los hijos de madre diabética. Los prematuros tienen Los gérmenes más frecuentemente implicados en las sepsis tardías
síntomas con más frecuencia que otros grupos de pacientes. Estas son: en las no nosocomiales, Streptococcus agalactiae, E. coli y Listeria;
manifestaciones clínicas son variables y pueden confundirse con otras dentro de las de ámbito nosocomial, destacan S aureus y P. aeruginosa
patologías: temblores, apatía, convulsiones, crisis de apnea o por el (respuesta 2 falsa).

CTO Medicina • C/Francisco Silvela, 106 • 28002 Madrid • Tfno.: (0034) 91 782 43 30/33/34 • E-mail: secretaria@ctomedicina.com • www.grupocto.es
5
Pediatría
Test 2V Distancia

Los serotipos con factores de adhesión a las meninges de Streptococcus


Comentarios
Grupo CTO
CTO Medicina

La opción 4 es falsa, pues el crecimiento intrauterino es indepen-


agalactiae (serotipo III) y E. coli (serotipo K1) son los más frecuentemente diente de GH; por este motivo, los RN con déficit de GH no tienen
implicados en las meningitis neonatales (respuesta 3 correcta). Streptococ- por qué nacer con peso y talla bajos.
cus agalactiae y E. Coli son las bacterias más frecuentemente implicadas
en la sepsis neonatal de inicio precoz (respuesta 4 falsa). Pregunta 28.- R: 1
La enfermedad hemorrágica del RN es una grave entidad pato-
El antibiótico más eficaz frente a Listeria es la ampicilina (respuesta génica. La causa es un déficit de vitamina K que produce a su vez,
5 falsa); no ha de faltar por ello en el tratamiento empírico de toda déficit de factores K-dependientes. Es importante recordar que los
sepsis neonatal precoz. hijos de madres que durante el embarazo han tomado fenitoína o
fenobarbital tienen más riesgo de padecer este trastorno (respuesta
Pregunta 25.- R: 1 2 correcta), y también es más frecuente en niños portadores de coa-
Para diagnosticar una sepsis neonatal has de sospecharla ante gulopatía. La enfermedad de presentación tardía puede obedecer
cualquier neonato con factores de riesgo (fiebre materna intraparto, a malabsorción o a defectos hepáticos (respuesta 5 correcta). Por
datos de corioamnionitis, bolsa rota prolongada, madre portadora de esta razón, está indicado hacer un estudio completo de coagulación
estreptococo grupo B) y clínica inespecífica (palidez, letargia, llanto (respuesta 3 correcta). Su expresividad clínica es muy variada, desde
agudo, inestabilidad vasomotora, rechazo de las tomas, taquipnea, hemorragia digestiva hasta hemorragia intracraneal, pasando por un
distrés respiratorio) junto con hemograma que muestre leucopenia o abrupto cuadro de CID (respuesta 4 correcta).
leucocitosis extrema, neutropenia o desviación izquierda.
La opción 1 es incorrecta. Para tratar la enfermedad hemorrágica
Recuerda otras peculiaridades: del RN, hay que emplear vitamina K por vía IV (la dosis profiláctica
• Las manifestaciones neurológicas son más frecuentes en las de 1 mg i.m. no es suficiente) y plasma fresco congelado.
sepsis tardías.
• Los agentes etiológicos más frecuentes son: SGB y E. coli. Pregunta 29.- R: 5
• La mortalidad es mayor en las sepsis precoces que en las tardías. El valor de la Hb en un RN es mayor que en el adulto. Recuerda los
• El tratamiento empírico debe ser, al menos 7 días. Consiste niveles de Hb en el RN a término y pretérmino.
en ampicilina + gentamicina (si no meningitis) o ampicilina +
cefalosporina 3.a generación (si existe sospecha de meningitis).
NIVELES DE
NIVELES MÍNIMOS
Pregunta 26.- R: 4 HEMOGLOBINA
DE HEMOGLOBINA
La madre que puede contagiar una hepatitis B al RN es aquella AL NACIMIENTO
que tiene el Ag +, (hepatitis crónica activa, hepatitis aguda o porta- RN a término 14 - 20 g/dl 9 - 11g/dl (8-2 semanas)
dora). Si además el Age es positivo, el riesgo asciende hasta un 90%.
RN pretérmino 12 - 18 g/dl 7 - 9 g/dl (6 semanas)
El momento de mayor riesgo es el momento del parto.
Pregunta 29. Hemoglobina al nacimiento.
La mayoría de las veces, la enfermedad en el neonato es asinto-
mática, pero tiene una alta probabilidad de evolucionar hacia una
forma crónica y sufrir degeneración maligna. La hemoglobina comienza a descender a las 48 horas de vida; el
descenso alcanza un valor máximo antes en un RNPT (6 semanas)
Para evitar este curso es fundamental realizar profilaxis con gam- que en los RNT (8 semanas). Esto es así porque los RNPT tienen
maglobulina específica en las 12 primeras horas de vida y primera menor disponibilidad de hierro para hacer la eritropoyesis, y por-
dosis de vacuna Si la profilaxis se realiza correctamente, podrá recibir que presentan menos sensibilidad a la EPO (respuesta 5 falsa).
lactancia materna. Posteriormente hay que seguir la pauta de vacu-
nación de VHB a los 2 y 6 meses. Una de las causas más frecuentes de anemia en el RN es la anemia
hemolítica por isoinmunización anti-AB0 (suele ser menos grave)
Pregunta 27.- R: 4 y anti-D (suele ser más grave).
La opción 1 es correcta, pues en el defecto global de hormonas
hipofisarias habrá un déficit de LH, con lo cual secundariamente habrá La detección de hematíes fetales en sangre materna se conoce
un déficit de testosterona, que motivará la aparición de un micropene; como test de Kleihauer-Betke, y sirve para valorar transfusiones
a su vez, los déficits de GH y ACTH asocian hipoglucemia. fetomaternas ocurridas intraútero.

La opción 2 es también correcta: los niveles bajos de PTH cursan En fases iniciales de una hemorragia aguda, al igual que en cual-
con hipocalcemia, que puede derivar en tetania (por ejemplo, en el quier paciente (independientemente de su edad), la hemoglobina
seno de un síndrome de DiGeorge). no se altera, y hemos de fiarnos del contexto clínico para valorar la
severidad de la misma.
La opción 3 es, a su vez, correcta. La hemorragia suprarrenal suele
aparecer en el marco de un parto traumático de nalgas. También hay Pregunta 30.- R: 4
casos de hemorragia suprarrenal fulminante surgidos en el contexto El hijo de esta pareja puede desarrollar tanto una isoinmuniza-
de una sepsis (síndrome de Waterhouse-Friederichsen). Indepen- ción anti-D (si el feto fuese Rh+) como una isoinmunización anti-A
dientemente de la causa que la origine, la insuficiencia suprarrenal (si el feto fuese A). Obviamente, nos preocupará más el posible
cursa con: hipoglucemia mantenida, vómitos, diarrea, deshidratación, desarrollo de una isoinmunización anti-D. Esta isoinmunización
hiponatremia e hiperpotasemia (opción 5, correcta). requiere una sensibilización previa (la isoinmunización anti-AB0

CTO Medicina • C/Francisco Silvela, 106 • 28002 Madrid • Tfno.: (0034) 91 782 43 30/33/34 • E-mail: secretaria@ctomedicina.com • www.grupocto.es
6
Comentarios
Grupo CTO
CTO Medicina

no la requiere), esto es, un primer embarazo o un primer aborto


Test 2V Distancia Pediatría

La neumonitis es la forma más característica de infección postnatal.


con un fruto de concepción Rh +. Para despistar este hecho, se
recurre al test de Coombs indirecto, que detecta anticuerpos anti-D Pregunta 33.- R: 4
en sangre materna: La supervivencia de los nacidos con muy bajo peso aumenta en
• Si el test de Coombs indirecto es negativo, implica que la madre un ambiente térmico neutro, que es el que necesitan para mantener
no está sensibilizada. La profilaxis se hará sólo en el caso de que una temperatura corporal de 36,5-37 oC. La temperatura de la incu-
el test de Coombs indirecto sea negativo (respuesta 1 correcta), badora no tiene un valor fijo, debe regularse según la temperatura
y consiste en administrar gammaglobulina en la semana 28 de del neonato (respuesta 1 falsa); cuanto más pequeño y más inmaduro
gestación y en las primeras 72 horas postparto si el RN es Rh+ sea el niño, más temperatura necesitará.
o se desconoce. A través de la profilaxis, se pretende evitar con
la sensibilización de la madre (para que no fabrique anticuerpos La alimentación del prematuro debe iniciarse de una forma cau-
anti-D por sí misma). telosa y gradual, y si el niño no tiene dificultad respiratoria ni otros
• Si el test de Coombs indirecto es positivo, implica una sensibiliza- signos patológicos, y tiene movimientos de succión, puede intentarse
ción previa, con lo cual la prevención con Ig anti-D resulta inútil la alimentación vía oral (respuesta 2 falsa).
(respuesta 4 falsa).
Aunque los prematuros tienen riesgo aumentado de lesión hi-
Ante cuadros muy severos de isoinmunización anti-D, puede póxica y de insuficiencia respiratoria, la administración de oxígeno
llegar a ser necesario la transfusión de hematíes al feto (opción 2, conlleva riesgo de toxicidad pulmonar y ocular, por lo que la oxi-
correcta). Para valorar el grado de hemólisis intraútero, se recurre al genoterapia se administrará solo a los prematuros que la necesiten
análisis espectrofotométrico del líquido amniótico. La zona III implica (respuesta 3 falsa).
un alto riesgo fetal (respuesta 5 correcta). Las zonas I y II implican
menor riesgo. La leche materna, siempre que no haya contraindicaciones
específicas, es la preferida para la alimentación del lactante (res-
Pregunta 31.- R: 3 puesta 4 correcta). En el caso de los prematuros se pueden añadir
La definición de policitemia neonatal viene condicionada por suplementos de proteínas, calcio y fósforo (respuesta 4 correcta),
un valor del hematocrito central o venoso mayor o igual a 65% y si no se dispone de leche materna, se administran fórmulas es-
y/o una hemoglobina mayor de 20 g/dl. El hematocrito capilar peciales de prematuros.
suele ser mayor que el central y no es fiable para el diagnóstico
(respuesta 3 falsa). Las necesidades de líquidos dependen de la edad gestacional, las
condiciones ambientales, y la existencia de enfermedades. Las pér-
La policitemia se asocia con diversas patologías: transfusión didas insensibles son mayores cuanto menor sea la edad gestacional
maternofetal o fetofetal, retraso en la ligadura del cordón, CIR (respuesta 5 falsa).
o bajo peso por insuficiencia placentaria (respuesta 5 correcta),
recién nacidos postérmino, hijo de madre diabética (respuesta 2 Pregunta 34.- R: 3
correcta), hiperplasia adrenal, síndrome de Down, trisomías 13 y En el ombligo del recién nacido debemos distinguir la presencia
18, hipotiroidismo. de alteraciones típicas:
• La persistencia del uraco y del conducto onfalomesentérico pro-
El problema de la policitemia es que provoca hiperviscosidad. Lo ducen secreción líquida por el ombligo, ácida en el primer caso,
más frecuente es que exista clínica de temblores (por hipoglucemia y alcalina en el segundo.
e hipocalcemia asociadas), letargia, rechazo de alimento, taquipnea, • El granuloma (el caso clínico corresponde a ello), y el pólipo umbi-
priapismo e ictericia (respuesta 1 correcta). Este evento también puede lical aparecen como un tejido exuberante, una tumoración a nivel
derivar a insuficiencia cardíaca por enlentecimiento de la circulación del ombligo, no dolorosa, en el primer caso rosada, blanda y con
(respuesta 4 correcta). El signo clínico más característico es la plétora secreción serosa, y en el último, roja oscura, dura y con secreción
en mucosas, palmas y plantas. mucoide. El diagnóstico es clínico, realizándose biopsia si hay mu-
chas dudas, siendo el tratamiento del granuloma la cauterización,
El tratamiento consiste en control estrecho de glucemia, diuresis y el del pólipo la extirpación completa.
y tolerancia digestiva y en casos graves exanguinotransfusión parcial • La hernia umbilical se produce por un cierre imperfecto o debili-
por vena umbilical. dad del anillo umbilical, produciendo una protrusión de la pared
abdominal que contiene epiplon, fácilmente reductible y que la
Pregunta 32.- R: 5 mayoría de las veces se resuelve espontáneamente. Si no lo hace,
Es importante que recuerdes que el citomegalovirus es la causa o la hernia es muy importante, se cierra quirúrgicamente.
más frecuente de infección congénita. Si la infección se adquiere en • Las infecciones umbilicales en el período neonatal tiene riesgo
la primera mitad del embarazo, aparece un cuadro de coriorretinitis, de diseminación hematógena, al hígado o al peritoneo, y hay
calcificaciones periventriculares (en el caso de la toxoplasmosis son que tratarlas siempre agresivamente, con antibioterapia IV. Se
difusas) y microcefalia (respuesta 5 correcta). manifiestan por eritema periumbilical, con secreción purulenta
maloliente en algunos casos, y signos de infección sistémica en
Es más frecuente que la infección se contraiga en la segunda mitad casos avanzados.
de la gestación. En este caso, suele cursar de forma asintomática (90%
de casos), pero si tienen clínica suelen presentar ictericia, hepatoes- Pregunta 35.- R: 3
plenomegalia, calcificaciones, hepatitis. La secuela más frecuente es Muy importante recordar:
la sordera neurosensorial bilateral. • Características de la ictericia fisiológica vs no fisiológica:

CTO Medicina • C/Francisco Silvela, 106 • 28002 Madrid • Tfno.: (0034) 91 782 43 30/33/34 • E-mail: secretaria@ctomedicina.com • www.grupocto.es
7
Pediatría
Test 2V Distancia

- Inicio en las primeras 24 horas: SIEMPRE PATOLÓGICO (res-


Comentarios
Grupo CTO
CTO Medicina

Recuerda las principales características que las diferencian.


puesta 3 falsa).
- Predominio de bilirrubina directa: SIEMPRE PATOLÓGICO.
LECHE HUMANA LECHE DE VACA
• Otros datos de no fisiológica:
- Incremento mayor de 5 mg/24 horas. Calorías 670 kcal/l 670 kcal/l
- Bilirrubina en sangre de cordón mayor de 3 mg/dl. 1-1,5 g 3-4,5 g
- RNT >12 mg/dl y RNPT > 14 mg/dl. Proteínas Caseína 30% Caseína 80%
- Duración mayor de 14 días. Seroproteínas 70% Seroproteínas 20%

Hidratos de carbono 7 g lactosa y otras 4,5 g lactosa


Pregunta 36.- R: 4
3,5 g % ác. grasos
El hipotiroidismo congénito es una patología relativamente común esenciales
3,5 g
que se debe descartar en todos los neonatos. Su causa más frecuente Grasas Ác. grasos cadena larga
Escasos ác. grasos
esenciales
es la disgenesia tiroidea. Hoy en día se realiza el cribado neonatal insaturados
Ác. grasos saturados
determinando los valores de TSH en sangre obtenida entre los 2 y Colesterol
los 5 días de vida. Si los valores de TSH están elevados hay que sos- Minerales + +++ (5 veces)
pechar hipotiroidismo congénito e iniciar cuanto antes tratamiento
Hierro + +
sustitutivo con hormona tiroidea para evitar la aparición de retraso
mental irreversible. Cobre ++ +

Flúor + -
En el recién nacido los signos y síntomas de hipotiroidismo son
muy sutiles y la clínica se va instaurando de forma progresiva. Aparece Relación
2 1
facies peculiar, estreñimiento, ictericia prolongada, letargia, hernia calcio/fósforo
umbilical, fontanelas amplias, retraso en la maduración ósea y puede Vitamina A ++ +
asociarse una sordera neurosensorial.
Vitamina B + ++

Pregunta 37.- R: 3 Vitamina C + Escasa


Recuerda otro de los “retratos robot” de las múltiples enfermedades
Vitamina D + Escasa
que te pueden preguntar en el MIR:
• Decaimiento, hipoactividad. Vitamina E ++ +
• Cara empastada, poco expresiva.
Vitamina K Escasa +
• Macroglosia.
• Ictericia. Nitrógeno
+++ +
no proteico
• Hernia umbilical.
• Fontanelas aumentadas de tamaño (el neonato del enunciado Pregunta 38. Comparación entre la leche humana y la leche de vaca.
tiene una fontanela anterior normal, pero la fontanela posterior
es amplia).
Pregunta 39.- R: 5
Este es el cuadro típico del hipotiroidismo congénito.
• El hipertiroidismo no comparte ningún síntoma con el hipoti- TALLA RETRASO CONSTITUCIONAL
roidismo. BAJA FAMILIAR DEL CRECIMIENTO
• La fenilcetonuria cursa con decaimiento, pero no con ictericia,
Menor de la
macroglosia ni hernia. Talla RN
normal
Normal
• El raquitismo comparte con el hipotiroidismo tan sólo las fon-
tanelas aumentadas de tamaño. Velocidad
de Normal Menor de lo normal, luego normal
• La hiperplasia suprarrenal tampoco comparte ningún síntoma
crecimiento
con el hipotiroidismo.
Antecedentes
De talla baja De pubertad retrasada
familiares
Pregunta 38.- R: 5
• La leche de madre y la leche de vaca tienen la misma cantidad · Igual a edad talla
Igual a edad
de grasa (respuesta 1 falsa) pero ésta es cualitativamente distinta. Edad ósea
cronológica
· Retrasada respecto a la edad
La leche de madre presenta más colesterol, ácidos grasos esen- cronológica
ciales y ácidos grasos poliinsaturados de cadena larga (respuesta Talla final Disminuida Normal o algo disminuida
5 correcta).
• El flúor es más abundante en la leche materna que en la de vaca Pregunta 39. Talla baja familiar versus retraso constitucional
(respuesta 2 falsa). del crecimiento.
• La leche de madre presenta una relación caseína/seroproteínas
de 30/70. La de vaca, de 80/20. Recuerda que la causa más frecuente de talla baja en el niño son
• La leche de vaca presenta más cantidad de vitamina K (respuesta las dos variantes de la normalidad.
4 falsa) que la leche de madre (por ello, los alimentados con fór-
mula adaptada presentan menor riesgo de desarrollar enfermedad Ante un caso de talla baja familiar se objetiva una curva de
hemorrágica del RN). crecimiento igual o inferior al percentil 3, sin discordancia entre la

CTO Medicina • C/Francisco Silvela, 106 • 28002 Madrid • Tfno.: (0034) 91 782 43 30/33/34 • E-mail: secretaria@ctomedicina.com • www.grupocto.es
8
Comentarios
Grupo CTO
CTO Medicina

edad ósea del niño y su edad cronológica. Además existe historia


Test 2V Distancia Pediatría

En la TBF la edad ósea coincide con la cronológica (respuesta 2


familiar de talla baja. La pubertad se produce a la edad habitual falsa). En el RCCP la edad ósea es menor que la cronológica (respuesta
y la talla adulta final es baja, pero dentro de los límites de lo 3 correcta).
esperado para su talla genética. Todas las pruebas de laboratorio
son normales. En el RCCP la talla final es normal o levemente disminuida (res-
puesta 4 correcta).
Pregunta 40.- R: 2
Caso clínico que presenta a un lactante con diarrea florida (piensa En la TBF la pubertad no aparece retrasada (respuesta 5 correcta).
en la posibilidad etiológica de Rotavirus), con signos clínicos de des-
hidratación intracelular (importante sequedad de piel y mucosas e Pregunta 43.- R: 2
hiperexcitabilidad neurológica); acidosis metabólica (pH<7,35 con Recuerda que, para el MIR, has de tener muy en cuenta las dife-
bicarbonato<21 mEq/l), e hipernatremia (Na>145 mEq/l). Se trata rencias entre:
de una deshidratación hipertónica. • Laringotraqueítis vírica: cuadro de disfonía, estridor, tos pe-
rruna, dificultad respiratoria leve-moderada, empeoramiento
Toda deshidratación hipernatrémica debe corregirse en 48-72 nocturno y fiebre, de etiología viral (virus parainfluenzae en el
horas, pues una corrección rápida de la misma podría derivar en 75% de los casos) caracterizado por un antecedente previo de
la aparición de edema cerebral (respuesta 2 correcta). infección de vías altas.
Afecta a niños entre 3 meses y 5 años y tiene predominio
Recuerda que la deshidratación isotónica es la más frecuente en estacional produciéndose la mayoría de las veces en otoño. El
los países desarrollados. Repasa los tres tipos: tratamiento consiste en humedad ambiental, corticoides (en
aerosol o sistémicos) y adrenalina en aerosol.
• Laringitis espasmódica: igual que la anterior pero sin fiebre.
ISOTÓNICA HIPOTÓNICA HIPERTÓNICA
• Traqueítis bacteriana: laringitis que progresa torpemente,
con empeoramiento del estado general, fiebre alta y mayor
Pérdida Agua = solutos Solutos > agua Agua > solutos dificultad respiratoria.
• Epiglotitis: comienzo súbito, rápidamente progresivo, con
Osmolaridad 285 mosm/l < 270 mosm/l > 300 mosm/l
mal estado general, importante dificultad respiratoria, dislalia
Na 130-150 mEq/l < 130 mEq/l > 150 mEq/l en patata caliente, disfagia e hiperextensión cervical.

· Mucosas · Mucosas secas +


· Mucosas secas Pregunta 44.- R: 4
secas · Pliegue ++
++ Se trata de un caso típico de traqueítis bacteriana. Hay que sospe-
· Pliegue + · Fontanela deprimi-
Clínica · Fontanela da +
· Sed char esta entidad cuando, tras un cuadro de crup vírico, se produce
· NRL
deprimida · Hipotensión + un empeoramiento progresivo con fiebre, dificultad respiratoria de
(hemorragia
· Hipotensión · Oliguria +
subdural) intensidad creciente y estridor mixto. Es más frecuente en niños me-
· Oliguria · Convulsiones
nores de 3 años y está producida en la mayoría de las ocasiones por
Pregunta 40. Tipos de deshidratación.
Staphylococcus aureus.

El diagnóstico es clínico, sin que haya alteraciones analíticas ni


Pregunta 41.- R: 1 radiológicas específicas.
En el raquitismo las primeras alteraciones se dan en el plano ra-
diológico (respuesta 2 correcta): pobre mineralización ósea, metáfisis El tratamiento de elección son los antimicrobianos antiestafilocócicos
en copa (respuesta 4 correcta), incurvación de los huesos largos, zonas (cloxacilina, amoxicilina-clavulánico; vancomicina si se sospecha una
de Looser-Milkmann. cepa meticilín-resistente). La intubación puede ser necesaria en los
casos graves, pero no se realiza de rutina.
Las necesidades diarias de vitamina D para un RN normal son 400
UI (respuesta 3 correcta). Aunque estamos en un país suficientemente El pronóstico es bueno, mejorando la mayoría notablemente tras 2-3
soleado, prácticamente todos los pediatras pautan para sus lactantes días de tratamiento antibiótico. Como otras infecciones que produce
vitamina D desde los primeros 15-30 días hasta el año de vida, si bien Staphylococcus aureus, puede ir asociada a un síndrome de shock tóxico,
a título teórico esta actitud no sería necesaria. patología producida por la toxina de esta bacteria.

Los niños de raza negra padecen más raquitismo carencial que Pregunta 45.- R: 1
los de raza blanca, por motivos esencialmente socioeconómicos La bronquiolitis es una enfermedad frecuente de las vías respi-
(respuesta 5 correcta). ratorias, que afecta a niños menores de 2 años de edad, especial-
mente a lactantes menores de 6 meses, que se presenta de forma
La existencia de craneotabes parietal es fisiológica en los RN, no estacional, con un pico en invierno, y que plantea el diagnóstico
así la occipital, considerada patológica siempre (respuesta 1 falsa). diferencial principalmente con la hiperreactividad bronquial. Los
antecedentes familiares de asma, el comienzo brusco de los síntomas
Pregunta 42.- R: 2 no precedido de infección, la eosinofilia, la repetición de episodios
En el retraso constitucional del crecimiento y la pubertad, al igual similares, apoyan el diagnóstico de broncoespasmo. En la radiogra-
que en la talla baja familiar, los niveles de GH son normales (respuesta fía de tórax en ambos casos podemos encontrar hiperinsuflación
1 correcta). pulmonar, siendo frecuente además en los casos de bronquiolitis

CTO Medicina • C/Francisco Silvela, 106 • 28002 Madrid • Tfno.: (0034) 91 782 43 30/33/34 • E-mail: secretaria@ctomedicina.com • www.grupocto.es
9
Pediatría
Test 2V Distancia

la presencia de infiltrados pulmonares y áreas de consolidación


Comentarios
Grupo CTO
CTO Medicina

En nuestro paciente, sin desencadenante alguno (así queda descar-


por atelectasias. tada la aspiración de cuerpo extraño al árbol bronquial, cuadro que
entraría en el diagnóstico diferencial de la tos súbita con sibilancias,
Mientras que el tratamiento del broncoespasmo son los bronco- y que tiene una primera fase de sofocación), aparece tos, dificultad
dilatadores (beta-2 agonistas, anticolinérgicos), y los corticoides en los respiratoria, sibilancias y signos radiológicos de atrapamiento aéreo.
casos moderados-graves, ninguno de ellos ha demostrado beneficio en
la bronquiolitis. Lo más importante en el tratamiento de la bronquiolitis La opción 1 es falsa, pues el paciente tiene más de dos años. La
son las medidas físicas y la oxigenoterapia cuando la saturación del opción 2 es falsa, pues la tos psicógena cursa sin sibilancias y sin
niño no es la adecuada. radiología de atrapamiento. La opción 4 es falsa, pues el paciente
está afebril, y no tiene infiltrados intersticiales en la placa de tórax. La
Pregunta 46.- R: 4 opción 5 es falsa, pues el crup suele afectar a niños más pequeños,
Recuerda algunos datos relacionados con la bronquiolitis. presenta estridor inspiratorio y no cursa con sibilancias.
• El adenovirus provoca una bronquiolitis especial, conocida
como síndrome del pulmón hiperclaro unilateral (esto lo han Pregunta 49.- R: 2
preguntado en el MIR). Caso clínico típico de epiglotitis aguda que se debe sospechar
• El VRS provoca infecciones respiratorias inocentes en adultos y por la presencia de fiebre alta, dolor de garganta, babeo y dificultad
niños mayores, muchos de los cuales actúan como “vector” de respiratoria en un niño que prefiere estar sentado e inclinado hacia
la infección sobre los lactantes. delante. Actualmente los principales patógenos responsables del cua-
• La exploración de un lactante con bronquiolitis muestra durante dro son cocos grampositivos (S. pyogenes, S. pneumoniae y S. aureus),
la inspección signos de dificultad respiratoria (tiraje supraes- por detrás de éstos se encuentra el agente clásico, el H. tipo B (cada
ternal, subcostal e intercostal), y a la auscultación, polipnea, vez menos frecuente gracias a la vacunación universal).
espiración alargada, sibilancias, crepitantes espiratorios e hi-
poventilación. El diagnóstico es clínico pero si fuera necesario se realizaría una Rx
• Es raro que un lactante con bronquiolitis se sobreinfecte por lateral de laringe donde se podría visualizar una epiglotis engrosada.
bacterias. Es un cuadro muy grave donde suele ser necesario establecer una
• Un 20% de los lactantes que padecieron bronquiolitis presentará en vía aérea artificial a través de la intubación nasotraqueal (respuesta 2
un futuro hiperreactividad de las vías aéreas durante la infancia. falsa) siempre en condiciones de seguridad (UCI, quirófano), junto con
administración de cefalosporinas de 3.a generación. Los corticoides
Pregunta 47.- R: 5 pueden ser beneficiosos. Con el tratamiento adecuado, suele remitir
Este caso clínico te puede poner en la encrucijada de dos diagnós- en 24-48 horas.
ticos diferenciales: epiglotitis aguda y absceso retrofaríngeo. Podría ser
una epiglotitis, por la conjunción de dificultad respiratoria, fiebre alta En el curso de la epiglotitis pueden aparecer complicaciones como
y disfagia. Pero en las epiglotitis no se ve ninguna masa en la pared neumonía, linfadenitis cervical y otitis.
posterior de la faringe.
Pregunta 50.- R: 2
En el absceso retrofaríngeo (respuesta 5 correcta) afecta fun- La fibrosis quística es una enfermedad multisistémica caracte-
damentalmente a menores de 5 años y casi siempre supone una rizada fundamentalmente por obstrucción e infección de las vías
complicación de una faringoamigdalitis por extensión de la infección respiratorias y por mala digestión. En su patogenia está implicada una
a los ganglios situados entre la pared posterior de la faringe y la fas- alteración de la regulación de los canales iónicos de las membranas
cia paravertebral. Suele iniciarse de manera brusca con fiebre alta, celulares, lo que produce secreciones deshidratadas y espesas. Se
disfagia, rechazo de alimento y rigidez cervical. En la exploración de hereda de forma autosómica recesiva, y es más frecuente en la raza
observa abombamiento de la pared posterior de la faringe. caucásica. Las mutaciones que causan la enfermedad son diversas,
pero todas afectan al mismo locus del brazo largo del cromosoma 7; en
Descartadas quedarían, de entrada, el resto de las opciones: Europa, el 50% de los pacientes aproximadamente son homocigotos
• Las faringoamigdalitis aisladas no provocan distrés ni asocian para la deleción F508.
masa retrofaríngea.
• La sofocación por cuerpo extraño no provoca fiebre. La fibrosis quística es la causa más frecuente de neumopatía cró-
• En la traqueítis bacteriana no hay disfagia ni masa faríngea. nica en la infancia y de insuficiencia pancreática exocrina, y además
produce otras alteraciones como síndrome pierde sal, poliposis nasal,
Pregunta 48.- R: 3 pansinusitis, prolapso rectal, pancreatitis, colelitiasis, diabetes mellitus.
La dificultad respiratoria infantil se puede clasificar conceptual-
mente en dos tipos: La aspergilosis broncopulmonar alérgica es relativamente frecuente
• Inspiratoria. Bajo ella subyace patología laríngea. Su elemento en estos pacientes. Hay que sospecharla ante la presencia de esputo
auscultatorio clave es el estridor inspiratorio. herrumbroso, aislamiento de Aspergillus fumigatus o la presencia
• Espiratoria. Bajo ella subyace patología del árbol bronquial. Su de eosinófilos en una muestra fresca de esputo. El tratamiento será
fenómeno auscultatorio clave es la sibilancia. El primer episodio corticoterapia oral, en los casos refractarios puede ser necesario el
de dificultad respiratoria sibilante en pacientes de menos de dos empleo de anfotericina B en aerosol o de 5-fluorocitosina sistémica.
años se define como bronquiolitis; la recurrencia de episodios
sibilantes se conoce como asma (obstrucción reversible de la vía El objetivo principal del tratamiento es mantener al paciente esta-
aérea asociada a inflamación y broncoespasmo). El asma infantil ble, sin reagudizaciones de la enfermedad, el mayor tiempo posible,
puede obedecer a procesos infecciosos o a fenómenos de alergia. siendo la terapia pulmonar y nutricional lo más importante. La primera

CTO Medicina • C/Francisco Silvela, 106 • 28002 Madrid • Tfno.: (0034) 91 782 43 30/33/34 • E-mail: secretaria@ctomedicina.com • www.grupocto.es
10
Comentarios
Grupo CTO
CTO Medicina

se fundamenta en la antibioterapia, oral y nebulizada, siendo muy


Test 2V Distancia

Pregunta 53.- R: 4
Pediatría

importante la identificación de los microorganismos que producen En relación con la invaginación intestinal recuerda:
las exacerbaciones, y ajustar en lo posible el espectro antibacteriano • Hay que descartar que se organice sobre una lesión orgánica
al resultado de los antibiogramas. Cuando el paciente desarrolla previa, como el linfoma, sobre todo si el paciente tiene más de
síntomas progresivos o no mejora a pesar de la antibioterapia oral y seis años.
nebulizada, está indicada la intravenosa, siendo frecuente su admi- • Si se deja evolucionar una invaginación, con el tiempo aparecerá
nistración domiciliaria en caso de que el paciente no precise otros compromiso de irrigación, necrosis y deposiciones hemorrágicas.
cuidados hospitalarios. • Se suele diagnosticar a través de ecografía abdominal.
• Sólo un 5-8% de los casos tratados con enema recidivan (res-
Pregunta 51.- R: 3 puesta 4 falsa). De los tratados con cirugía recidivan un 3%.
En un neonato que presenta íleo meconial debemos sospechar • El signo del muelle enrollado en el enema opaco apoya el
fibrosis quística. diagnóstico.

El estudio molecular genético directo sería el método diagnóstico Pregunta 54.- R: 2


principal en este caso (respuesta 3 correcta). La existencia de dos El diagnóstico es el de atresia esofágica con fístula traqueoesofágica
mutaciones conocidas, en el brazo largo del cromosoma 7, confir- distal (tipo III según la clasificación de Ladd), el tipo más frecuente (más
marían el diagnóstico. del 85%) de todos los casos. En los casos en que hay fístula proximal,
el abdomen está excavado, porque no llega al tubo digestivo aire, ni
El test del tripsinógeno inmunorreactivo sérico (que estará elevado por el esófago, ni por la vía respiratoria y pueden deglutir, pero cada
si hay afectación pancreática) serviría para el cribado neonatal, pero vez que lo hacen tienen un episodio de aspiración grave.
no para establecer el diagnóstico definitivo.
El diagnóstico se realiza por la sospecha clínica (salivación excesiva,
La prueba del sudor serviría para el diagnóstico, pero no en neo- cianosis y atragantamiento con las tomas, etc.) e imposibilidad para
natos pues en las primeras semanas de vida es poco fiable, por la pasar una sonda nasogástrica y generalmente la radiografía de tórax
dificultad de obtener sudor. es suficiente para confirmarlo, viéndose la sonda enrollada en la bolsa
esofágica superior. Es importante el antecedente de polihidramnios
Recuerda que para el diagnóstico de fibrosis quística se requiere por imposibilidad del feto para deglutir líquido amniótico. El 30%
manifestaciones clínicas compatibles y/o antecedente de hermano/a de los afectados nacen prematuros y el 50% tienen malformaciones
con enfermedad confirmada, y/o cribado neonatal positivo más asociadas (recordad el acrónimo VACTERL: vertebrales, anorrectales,
un criterio de confirmación (test del sudor positivo o presencia de cardíacas, traqueales, esofágicas, renales y del radio).
mutaciones en el estudio genético o test de diferencia de potencial
nasal positivo). El tratamiento es quirúrgico. El reflujo gastroesofágico es la compli-
cación postquirúrgica más frecuente y suele ser grave. Otras compli-
Pregunta 52.- R: 5 caciones menos frecuentes son la fístula de la anastomosis, la recidiva
Este es el “retrato robot” de la invaginación intestinal: de la fístula traqueoesofágica, la estenosis esofágica, la traqueomalacia
• A un lactante. y la lentitud para alimentarse y, como consecuencia, fallo de medro.
• Con llanto paroxístico.
• Signos localizadores abdominales (encogimiento de piernas Pregunta 55.- R: 3
hacia el vientre). Este es el “retrato robot” de la enfermedad celíaca: niño pequeño
• Signos indirectos de sufrimiento (palidez). con deposiciones de características esteatorreicas, anorexia, retraso
• Y masa en hemiabdomen derecho. ponderal, mal talante, anemia ferropénica (ferritina y sideremia bajas)
• Si la clínica evoluciona, hasta un 60% puede presentar depo- y test de Van de Kammer con aumento de la grasa total.
siciones en “jalea de grosella”, heces con sangre roja fresca y
moco. En el año 2012 la ESPGHAN publicó las nuevas directrices para el
diagnóstico de la enfermedad celiaca. Por ello, merece la pena que
Debes recordar: recordemos algunas consideraciones en el diagnóstico de la misma y
• La invaginación intestinal es la causa más frecuente de obstruc- puntualicemos cuáles han sido los cambios introducidos.
ción intestinal entre los 3 meses y los 6 años de vida.
• Sólo en contadas ocasiones se logra advertir una causa orgánica El diagnóstico se basa en tres pilares: los anticuerpos, la genética
que lo favorezca (adenitis mesentérica, linfoma, etc.). y la biopsia intestinal.
• En la radiología se verá un silencio aéreo en hemiabdomen
derecho pero es la ecografía abdominal la técnica diagnóstica • Anticuerpos.
de elección (imagen de ¨donut¨). - IgA-antitransglutaminasa: muy sensibles y específicos, por
• Una invaginación intestinal es siempre una urgencia. Si han lo que constituyen el marcador serológico de elección para
trascurrido menos de 48 horas y no hay signos de perforación, el cribado. Recuerda que debemos solicitar recuento de IgA
se realiza tratamiento con enemas de bario o con aire para sérica total para descartar un posible déficit de IgA asociado
intentar la reducción. Si hay perforación intestinal o distensión (en cuyo caso solicitaremos la modalidad Ig G).
abdominal de más de 48 horas de evolución, se prefiere trata- - Antiendomisio: los más específicos, pero algo menos sen-
miento quirúrgico. sibles y su determinación no está automatizada.
• La localización más frecuente de la invaginación es la íleocólica, - Antigliadina: muy poco específicos, ya que pueden elevarse
seguida de la ileoileocólica. en otros procesos intestinales.

CTO Medicina • C/Francisco Silvela, 106 • 28002 Madrid • Tfno.: (0034) 91 782 43 30/33/34 • E-mail: secretaria@ctomedicina.com • www.grupocto.es
11
Pediatría
Test 2V Distancia

- Recientemente se han descubierto los anticuerpos antipép-


Comentarios
Grupo CTO
CTO Medicina

es clínico y que sólo en el RGE patológico está indicada la


tido desaminado de la gliadina, que son mejores que los realización de pruebas complementarias. El tránsito superior
antigliadina y son útiles en el diagnóstico de los niños me- con bario sería la primera prueba a realizar para descartar
nores de 2 años (en los que el resto de anticuerpos pueden alteraciones anatómicas.
ser negativos). Suele determinarse el tipo IgG. • En los casos leves, basta con establecer medidas dieteticopos-
turales (respuesta 4 correcta).
• Estudio genético. El 90% de los pacientes con enfermedad ce- • Los síntomas respiratorios incluyen: tos crónica, laringitis de
líaca son HLA-DQ2 positivos y el resto HLA-DQ8. La presencia repetición, bronquitis de repetición y neumonía por aspiración
de HLA compatible únicamente indica predisposición genética (respuesta 5 correcta).
pero no es diagnóstico de enfermedad (es un factor “necesario”,
aunque no suficiente). Es útil fundamentalmente para descartar Pregunta 59.- R: 4
enfermedad (tiene un alto valor predictivo negativo), ya que si Has de tener en cuenta que estreñimiento que comienza
el estudio genético es negativo es muy raro que el individuo desde el nacimiento+retraso ponderoestatural+antecedente de
desarrolle la enfermedad. retraso en la evacuación de meconio es igual a enfermedad de
• Biopsia intestinal. Muestra cambios histológicos característicos Hirschprung.
(aunque no patognomónicos): infiltrado linfocitario (especial- • La alteración más típica del enema opaco de los pacientes con
mente de linfocitos T gamma-delta), hiperplasia de las criptas y enfermedad de Hischprung es la presencia de un recto cuyo
atrofia de las vellosidades. Actualmente, en algunas situaciones, calibre es la mitad del segmento previo a la estenosis y el retraso
se puede llegar al diagnóstico SIN biopsia (principal novedad en la eliminación de contraste.
de las guías actuales). Por el momento, el único supuesto en el • En la manometría, el evento más característico es la presencia
que se puede establecer el diagnóstico de enfermedad celiaca de una contracción del esfínter anal interno ante aumento de
SIN BIOPSIA es en el caso en que la clínica sea compatible, las presiones en territorios proximales a él.
los títulos de anticuerpos sean elevados (IgA antitransglu- • El diagnóstico definitivo lo da la biopsia rectal (ausencia de
taminasa>10 veces el límite superior de la normalidad y células ganglionares con aumento de las terminaciones nerviosas
anticuerpos antiendomisio positivos) y la genética sea HLA y de la actividad de la acetilcolinesterasa).
DQ2 y/o DQ8. • El tratamiento de elección es la cirugía con la resección de
todo el segmento agangliónico.
Pregunta 56.- R: 4
La biopsia intestinal en el caso de la enfermedad celíaca no es
patognomónica pero sí muy característica. Aparece: MEGACOLON
MEGACOLON FUNCIONAL
• Atrofia subtotal o total de las vellosidades. CONGÉNITO
• Hiperplasia de las criptas. Inicio > 2 años Neonatal
• Infiltrado linfocitario en la lámina propia.
Retraso peso Raro Frecuente

Pregunta 57.- R: 4 Encopresis Frecuente Rara


Los anticuerpos más sensibles y específicos son los antitrans- Enterocolitis No aparece Posible
glutaminasa tipo IgA. La positividad de los anticuerpos se co-
Distensión
rrelaciona con la actividad de la enfermedad, y se utiliza como abdominal
Rara Presente
monitor del curso de la enfermedad y como sensor de transgre-
Tacto rectal Heces en ampolla Ampolla vacía
siones dietéticas.
Radiología Heces abundantes Datos típicos
Pregunta 58.- R: 3 Manometría Relajación del esfínter Ausencia de relajación
Este caso clínico presenta:
Biopsia Normal Patológica
• A un lactante varón.
• Con síndrome de Down, Pregunta 59. Diagnóstico diferencial del estreñimiento.
• Síntomas respiratorios (tos nocturna),
• Y síndrome de Sandifer (hipertonía cervical tras las tomas).
Pregunta 60.- R: 2
Ante este cuadro hay que pensar en una enfermedad por RGE. Recuerda para el MIR: diarrea de curso crónico (con deposiciones
que contienen fibras vegetales sin digerir y moco), que no asocia
Pasemos a analizar las distintas respuestas: retraso ponderal es igual a diarrea crónica inespecífica.
• La esofagitis por RGE provoca microsangrados que derivan
en la aparición de anemia ferropénica (respuesta 1 correc- La diarrea crónica inespecífica es una diarrea inocente debida a
ta). un peristaltismo intestinal aumentado de etiología desconocida, que
• El RGE es muy frecuente (1/300 RN lo presentan), sobre todo ocurre en algunos niños entre los 6 meses y los 3-4 años de vida. Es un
en varones (respuesta 2 correcta). equivalente del colon irritable del adulto, con el cual comparte patoge-
• La técnica de elección en el diagnóstico del RGE es la pH- nia (origen motor) y familiaridad (los hijos de padres con colon irritable
metría de 24 horas (respuesta 3 falsa), que es la prueba más tienen mayor probabilidad de desarrollar diarrea crónica inespecífica).
sensible y específica para cuantificar el grado de reflujo. Se
realiza sobre todo cuando existen manifestaciones extradi- El diagnóstico es clínico, siendo todos los parámetros de laboratorio
gestivas del reflujo. No olvides que, aun así, el diagnóstico en sangre y en heces normales.

CTO Medicina • C/Francisco Silvela, 106 • 28002 Madrid • Tfno.: (0034) 91 782 43 30/33/34 • E-mail: secretaria@ctomedicina.com • www.grupocto.es
12
Comentarios
Grupo CTO
CTO Medicina

La actitud a seguir consiste en serenar a los padres, y explicarles el


Test 2V Distancia Pediatría

Aunque la mejoría durante los fines de semana y las vacaciones van


carácter autolimitado de este tipo de diarrea (respuesta 2 correcta). a favor del dolor abdominal funcional, este patrón puede estar ausente,
y el niño incluso puede despertarse por la noche quejándose del dolor.
Pregunta 61.- R: 5
Recuerda para el día del MIR que: lactante + manifestaciones alér- Habitualmente el dolor abdominal funcional se localiza en la región
gicas (habones, angioedema, broncoespasmo) + síntomas digestivos periumbilical y es difícil que el niño lo defina claramente. Salvo que los
(vómitos, diarrea, rectorragia) es igual a alergia a proteínas de leche de síntomas sugieran una enfermedad péptica, no está indicado realizar
vaca. El cuadro está mediado por un mecanismo inmunológico tipo IgE. pruebas de detección de H. pylori, pues la prevalencia de infección
Afecta a individuos atópicos o con antecedentes familiares de atopia. crónica por esta bacteria en los niños con dolor abdominal recurrente
El diagnóstico es fundamentalmente clínico, se basa en la exclusión de es la misma que en la población general.
dichas proteínas y consecuente desaparición de los síntomas y la poste-
rior provocación. Se emplean las pruebas cutáneas que serán positivas. Pregunta 65.- R: 5
El divertículo de Meckel es un resto del conducto onfalomesentéri-
La actitud terapéutica a seguir con estos pacientes es retirar la fór- co. Es la malformación digestiva más frecuente apareciendo en un 2%
mula adaptada que estaba tomando, y sustituirla por un hidrolizado de lactantes. Se localiza a unos 50-70 cm de la válvula ileocecal. La
de proteínas. Si el paciente tomaba leche materna, basta con retirar manifestación más frecuente del divertículo de Meckel es la hemorra-
de la dieta de la madre los lácteos. Suele ser un proceso autolimitado. gia indolora e intermitente. También puede provocar dolor abdominal
agudo y fenómenos de diverticulitis. Todos estos fenómenos se deben
Pregunta 62.- R: 4 a la presencia de mucosa gástrica ectópica, que genera un ambiente
En la diarrea por mala absorción de hidratos de carbono las de- ácido que erosiona la vecindad del divertículo.
posiciones son espumosas y ácidas (pH menor de 5,5), asocian dolor
abdominal, aumento de los ruidos hidroaéreos y provocan eritema La técnica diagnóstica más sensible es la gammagrafía con Tc-99
perianal. El Clinitest, que detecta la presencia de azúcares reductores (que detecta únicamente aquellos divertículos en los que hay ectopia
en heces, es positivo en la malabsorción de lactosa. gástrica). Para aumentar la validez de esta prueba se puede utilizar
fármacos u hormonas que son dianas para las células gástricas (cime-
El síndrome postenteritis es un déficit transitorio de lactasa secun- tidina, glucagón, gastrina).
dario a un proceso de gastroenteritis aguda (que lesiona el borde del
enterocito), y que supone la intolerancia a la lactosa. Este síndrome El tratamiento es quirúrgico.
se puede intentar evitar si se hidrata y se nutre precozmente con Pregunta 66.- R: 4
una dieta normal a los pacientes con GEA. El tratamiento consiste El reflujo gastroesofágico fisiológico es la modalidad de reflujo que
en la retirada de la lactosa de la dieta hasta la remisión del cuadro afecta a niños menores de 2 años, cursa con síntomas típicos (como
(respuesta 4 falsa). el vómito atónico o regurgitación tras las tomas) y no produce retraso
ponderoestatural. Este reflujo no complicado desaparece en más de la
Pregunta 63.- R: 4 mitad de los casos a los 2 años (respuesta 4 correcta) pues se adopta
Un varón que presenta desde la tercera semana de vida vómitos la posición erecta y se modifica la consistencia de los alimentos.
alimentarios (nunca biliosos), proyectivos tras todas las tomas, es igual a
estenosis hipertrófica de píloro. Recuerda que en algunos casos puede Es importante diferenciar el caso anterior de la enfermedad por
palparse una masa con forma de aceituna en epigastrio/hipocondrio reflujo gastroesofágico en la que, junto al reflujo, aparecen más
derecho (¨oliva pilórica¨). Es típica la alcalosis metabólica hipoclorémi- situaciones patológicas como: escasa ganancia ponderal, clínica
ca debido a las pérdidas de hidrogeniones y cloruros con los vómitos. respiratoria, esofagitis, etc.

La prueba diagnóstica de elección ante la sospecha de este cuadro Pregunta 67.- R: 3


sería una ecografía abdominal. El hallazgo frecuente en la radiografía Los datos que han de ponerte en el camino diagnóstico del sín-
simple de abdomen es la distensión gástrica con escaso gas distal. drome de Reye son:
El tránsito digestivo está en desuso en la actualidad, se reserva para • Niño en edad escolar.
pacientes en los que la ecografía no es concluyente donde aparece • Antecedente de varicela/gripe.
de forma típica la imagen del “signo de la cuerda”. • Consumo de AAS.
• Datos de encefalopatía (letargia, sonmolencia, vómitos).
El tratamiento es quirúrgico: pilorotomía extramucosa de Ramsted. • Datos de afectación hepática (hipertransaminasemia, hipera-
moniemia, hipoglucemia) sin ictericia.
Pregunta 64.- R: 4 • Aumento de la glutamato deshidrogenasa.
Se estima que hasta un 10% de los preescolares y los escolares
padecen “dolor abdominal recurrente”. En los menores de 2 años, Recuerda que es un cuadro de afectación multiorgánica en el que
casi siempre se debe a una causa orgánica, pero en los niños mayores, la lesión ocurre fundamentalmente a nivel mitocondrial. Su frecuencia
sólo en el 10% se encuentra una causa orgánica. Aunque siempre ha disminuido en los últimos años, por evitar el empleo de aspirina
hay que pensar en la posibilidad de que haya una patología orgánica en los niños afectos de varicela o gripe.
subyacente, en los casos en los que la anamnesis y la exploración física
sugieren un origen funcional (no orgánico) del dolor, no es necesario Pregunta 68.- R: 2
realizar pruebas complementarias. En estos casos es muy importante En relación con el síndrome de Reye debes recordar:
intentar tranquilizar a la familia y al niño para su tratamiento, siendo • Hay metabolopatías de la ruta de la beta-oxidación y del ciclo
de peor pronóstico que los de causa orgánica. de la urea que cursan con clínica similar (acidosis, hipoglucemia,

CTO Medicina • C/Francisco Silvela, 106 • 28002 Madrid • Tfno.: (0034) 91 782 43 30/33/34 • E-mail: secretaria@ctomedicina.com • www.grupocto.es
13
Pediatría
Test 2V Distancia

vómitos, letargia). Esto es así porque comparten la organela


Comentarios
Grupo CTO
CTO Medicina

bos de plaquetas con trombopenia de consumo y anemia


disfuncional: la mitocondria. microangiopática. El órgano más afectado en este cuadro es
• La afectación neurológica de cualquier metabolopatía es difusa, el riñón.
nunca focal (respuesta 2 falsa). • La biopsia renal sólo está indicada en casos de insuficiencia
• El síndrome de Reye aparece en algunos escolares con gripe/ renal prolongada o cuando ésta no coexiste con trombopenia.
varicela que usan como antipirético el AAS. • Se realiza tratamiento conservador de la IRA y alteraciones
• Las medidas antiedema (manitol, hiperventilación controlada, hidroelectrolíticas y si no consiguen controlarse estará indicada
coma barbitúrico, restricción de líquidos) son clave, junto con la diálisis (respuesta 4 falsa).
el adecuado aporte de glucosa, vitamina K y plasma fresco • La función renal se recupera en un 90% de los pacientes con
congelado. un tratamiento adecuado y las recidivas son infrecuentes.

Pregunta 69.- R: 1 Pregunta 72.- R: 4


La fimosis es la estrechez del prepucio que impide su descubri- No olvides que el RVU es la causa de hasta un 20% de las insufi-
miento. Se considera fisiológica hasta los 2 años de edad (respuesta 1 ciencias renales y la primera causa de HTA en la infancia.
correcta). En las adherencias el glande no se visualiza por completo,
pero sí es posible su descubrimiento. Recuerda que para el diagnóstico, la técnica de elección es la cis-
tografía miccional que permite clasificar el reflujo según intensidad,
Debes saber que la circuncisión en menores de un año está indi- grado de dilatación ureteral y deformidad calicial (grados I-V). Se
cada sólo en infecciones urinarias de repetición, y en caso de existir debe realizar también una ecografía renal para descartar anomalías
un orificio prepucial mínimo que obstaculiza la emisión de orina. Las estructurales y una gammagrafía renal, técnica de referencia para el
pomadas de corticoides pueden ser útiles en el tratamiento aunque diagnóstico de cicatrices renales.
su efectividad no es del 100%.
Los objetivos del tratamiento son prevenir la pielonefritis y la
Pregunta 70.- R: 5 lesión renal.
La obstrucción de la vía urinaria puede localizarse a cualquier ni-
vel desde el meato uretral hasta los cálices, dependiendo de ello sus El reflujo grado I y II, en el 80% de los casos desaparece de forma
efectos fisiopatológicos, así como de la edad de comienzo, la magnitud espontánea al madurar el niño y no precisan tratamiento.
de la obstrucción, y de su forma de instauración, aguda o crónica.
La profilaxis antibiótica a dosis básica es el pilar fundamental del
En los niños, lo más frecuente es que la obstrucción sea congénita y reflujo leve hasta que éste desaparezca.
puede manifestarse en la vida fetal. La primera causa, por frecuencia, es
la estenosis de la unión pielocalicial, con un amplio espectro de gra- En los grados IV y V es de esperar que las repercusiones morfoló-
vedad; sin embargo, en los varones, la causa más frecuente de uropatía gicas a nivel renal no desaparezcan y/o se lesione más el riñón, por lo
obstructiva grave (no sólo obstrucción), son las valvas de uretra posterior. que se optará por la cirugía sin demora, que puede ser endoscópica
o abierta con muy buenos resultados (respuesta 4 falsa).
En cuanto a la estenosis pielocalicial, si es moderada-leve, y el riñón
funciona bien, se recomienda una actitud expectante aunque con Pregunta 73.- R: 4
controles periódicos; en muchos casos hay una mejoría espontánea El reflujo vesicoureteral (RVU), paso retrógrado de orina desde la
con el tiempo. En los casos graves, hay que poner profilaxis antibiótica, vejiga a la pelvis renal, predispone a la infección del tracto urinario. La
y considerar el tratamiento quirúrgico, que consiste en una pieloplas- reacción inflamatoria desencadenada puede dar lugar a la formación
tia, y que tiene entre un 91 y 98% de éxitos. Las indicaciones son: de cicatrices. Si éstas son extensas, se afectará la función renal (neu-
hidronefrosis bilateral grave, tumoración abdominal (hidronefrosis ropatía por reflujo). Esta entidad es la causa más frecuente de HTA
importante), riñón único, disminución de la función del riñón afectado. en la infancia y subyace en el 30% de las ITU pediátricas.

Pregunta 71.- R: 4 Pregunta 74.- R: 5


Este caso clínico sirve para ilustrar el retrato robot del síndrome Caso clínico de escroto agudo que por las características del cuadro
hemolítico urémico: (dolor intenso, tumefacción escrotal, sin fiebre ni traumatismo previo
• Antecedentes de diarrea sanguinolenta (recuerda la asociación y ausencia de reflejo cremastérico) podemos catalogar de torsión
con E. coli O157:H7). testicular. El diagnóstico se basa fundamentalmente en la clínica y en
• Clínica de deterioro neurológico (somnolencia, letargia, estupor). caso de duda se puede realizar una eco-Doppler con disminución
• Fracaso renal (se eleva BUN, creatinina, urea). de flujo testicular.
• Anemia (recuerda que es microangiopática, y en la extensión
periférica se ven esquistocitos). Se debe realizar diagnóstico diferencial con la orquioepididimitis,
• Trombopenia (clínica de petequias, equimosis). Se da por con- que cursa con dolor más progresivo e insidioso acompañado de
sumo de plaquetas. síndrome miccional y fiebre con reflejo cremastérico presente, y una
eco-Doppler con aumento de vascularización.
Recuerda:
• El SHU es la causa más frecuente de insuficiencia renal aguda El tiempo de viabilidad del testículo torsionado es de 4-6 horas,
en niños. por lo que siempre se debe considerar una urgencia quirúrgica. El
• El patógeno produce unas toxinas que favorecen la lesión a tratamiento consiste en la reducción manual o fijación quirúrgica del
nivel endotelial capilar y arteriolar con la formación de trom- teste afecto y del contralateral.

CTO Medicina • C/Francisco Silvela, 106 • 28002 Madrid • Tfno.: (0034) 91 782 43 30/33/34 • E-mail: secretaria@ctomedicina.com • www.grupocto.es
14
Comentarios
Grupo CTO
CTO Medicina

La torsión neonatal se sospechará ante una masa testicular firme e


Test 2V Distancia Pediatría

de petequias generalizadas y buen estado general, sin adenopatías y sin


indolora en el examen neonatal. La piel escrotal suele estar equimótica visceromegalias. Suele existir antecedente de infección viral o vacuna-
y/o edematosa. Habitualmente la torsión se produce intraútero y el ción 1-3 semanas antes. Cursa con disminución del número de plaquetas
infarto es la regla (respuesta 5 falsa). El tratamiento es la extirpación y alteración del tiempo de hemorragia. La mayor parte de los casos
vía inguinal y fijación contralateral del teste. tienen una recuperación espontánea. Sin embargo, pueden aparecer
complicaciones hemorrágicas graves, como la hemorragia intracraneal.
Pregunta 75.- R: 3
Caso clínico compatible con un neuroblastoma (niño menor de 2 Su patogenia se debe a destrucción por mecanismo autoinmune
años que presenta masa abdominal dura a la palpación, que sobrepasa de plaquetas, produciéndose secuestro y destrucción de las mismas
la línea media). Recuerda que se asocia a hematoma palpebral y he- por el sistema mononuclear fagocítico, principalmente en el bazo. Es
patomegalia y entre los síndromes paraneoplásicos (que no modifican una situación de hiperesplenismo (aumento de función del bazo) sin
el pronóstico) figuran el opsoclonus-mioclonus y la diarrea secretora. aumentar su tamaño, es clave recordar que LA ESPLENOMEGALIA
• La edad inferior a 1 año es un factor de buen pronóstico (res- ES UN DATO EN CONTRA DE PTI (respuesta 2 falsa).
puesta 1 falsa).
• Es un tumor que puede metastatizar en hígado sin que ello El tratamiento se basa en medidas generales (transfusión de pla-
contraindique la cirugía (respuesta 2 falsa). quetas, si precisa) y tratamiento específico (corticoides, de elección;
• Siempre se debe hacer aspirado de médula ósea para descartar gammaglobulina, cuando se precise aumento rápido de plaquetas;
afectación de la misma (respuesta 3 correcta). inmunosupresores, si PTI crónica, esplenectomia, etc.).
• El tumor que se presenta asociado a hemihipertrofia no es el neuro-
blastoma (respuesta 4 falsa) sino el tumor de Wilms o nefroblastoma. Pregunta 79.- R: 3
• Se sigue empleando la gammagrafía con MIBG como método La anemia ferropénica es la enfermedad hematológica más
diagnóstico y en dosis más altas puede constituir un posible frecuente en la infancia, y es la primera sospecha diagnóstica en
tratamiento (respuesta 5 falsa). un niño con anemia microcítica sin otros indicadores de patologías
específicas. Los niños, deben absorber de la dieta 1 mg de hierro
Pregunta 76.- R: 5 diario para mantener un balance positivo, y poder hacer frente a
Aprovecha esta pregunta para repasar los factores pronósticos sus necesidades por el crecimiento y por la descamación celular.
del neuroblastoma: Normalmente se absorbe un 10% del hierro de los alimentos, con
• BUEN PRONÓSTICO: lo cual un niño debe ingerir de 8 a 10 mg de hierro al día, siendo
-La edad menor de un año. difícil conseguir esto con la dieta, sobre todo en los primeros años
• MAL PRONÓSTICO: de vida, en los cuales predominan los alimentos lácteos y otros
-La localización abdominal. pobres en hierro. Por eso, aunque siempre hay que buscar una po-
-La amplificación del oncogén myc. sible pérdida de sangre ante una anemia ferropénica, desde los 9 a
-Las metástasis óseas constituyen mal pronóstico. No así las de los 24 meses, la causa más frecuente es la pobre ingesta. Hay que
hígado, piel o médula ósea. puntualizar sin embargo que esta causa es rara en los primeros 4-6
meses, porque aunque la leche materna y la de vaca son pobres en
Es importante que sepas que los síndromes paraneoplásicos asocia- hierro, los recién nacidos a término de peso adecuado nacen con
dos a neuroblastoma (el opsoclonus-mioclonus y la diarrea secretora) depósitos que suplen la carencia alimentaria. El hierro de la leche
no modifican el pronóstico (respuesta 5 falsa). materna se absorbe mejor que el de la leche de vaca, y los niños
alimentados con lactancia materna tienen menos necesidades de
Pregunta 77.- R: 2 hierro de otros alimentos que los que lactan con leche artificial.
Pregunta complicada. Se describe a un niño macrosómico (“gordo”)
con antecedente de onfalocele (“las tripas fuera”), para que pienses Las manifestaciones clínicas del déficit de hierro son las mismas
en el síndrome de Beckwith-Wiedemann. Este síndrome se asocia que las producidas por otro tipo de anemias, y además las debidas
con el tumor de Wilms. Aunque no te acuerdes del síndrome debes a la carencia de hierro en sí: pagofagia (deseo de comer sustancias
pensar en este tumor ante descripción de masa abdominal indolora extrañas), disminución de la atención y las capacidades intelectuales
que no suele pasar línea media. (incluso sin anemia), irritabilidad, anorexia.

Recuerda algunos datos: El tratamiento consiste en la administración de suplementos de hierro


• Otros síntomas son la HTA, en el 60% de pacientes por com- en forma de sales ferrosas (6 mg/kg/día de hierro elemental) vía oral. La
presión arterial por el tumor y producción de renina y puede respuesta es rápida y ayuda a hacer el diagnóstico en los casos en que
haber hematuria en un 10-25% de casos. no está claro. El primer indicador de mejoría es la clínica en cuanto a
• El diagnóstico se basa en la TC y la PAAF, no aconsejándose la irritabilidad y apetito del niño, en menos de 24 horas incluso. A las 48
realización de biopsia (para no alterar el estadio clínico). horas aproximadamente se produce una hiperplasia eritroide que puede
• Las metástasis más frecuentes son las pulmonares. objetivarse haciendo un examen de sangre periférica. El aumento de las
• El tumor de Wilms también se asocia a aniridia, hemihipertofia cifras de hemoglobina puede aparecer desde 4 días después de iniciar
y alteraciones genitourinarias. el tratamiento, hasta 1 mes después, dependiendo del déficit de base.
• El estadio V corresponde al tumor bilateral.
Pregunta 80.- R: 1
Pregunta 78.- R: 2 De la oncología pediátrica debes recordar:
La púrpura trombótica idiopática (PTI) suele ser una enfermedad • Cáncer infantil más frecuente: leucemias. Representan el
infantil que típicamente aparece como un cuadro de comienzo brusco 30%, de ellas el 97% son agudas, la mayoría linfoblásticas.

CTO Medicina • C/Francisco Silvela, 106 • 28002 Madrid • Tfno.: (0034) 91 782 43 30/33/34 • E-mail: secretaria@ctomedicina.com • www.grupocto.es
15
Pediatría
Test 2V Distancia

Los linfomas siguen a las leucemias en frecuencia, salvando


Comentarios
Grupo CTO
CTO Medicina

3-4 meses, aunque es excepcional que se prolonguen más de


el grupo de edad de entre 10 y 14 años, en que superan a las 1 año.
anteriores. • Artritis de rodillas y tobillos.
• Cáncer sólido infantil más frecuente: tumores del SNC, de ellos • Dolor abdominal cólico, vómitos y deposiciones con sangre.
el astrocitoma cuya localización más frecuente es infratentorial • Afectación renal: hematuria con o sin proteinuria. Es el factor
(fosa posterior). que marca el pronóstico.
• Tumor sólido extracraneal más frecuente: neuroblastoma. Se • Sólo en menos de un 1% se produce afectación del SNC.
localiza principalmente en abdomen. El neuroblastoma es más
frecuente en varones y en la raza blanca, y el 90% se diagnos- El tratamiento consiste en antiinflamatorios para las molestias
tican por debajo de los 5 años de edad. articulares y ciclos cortos de corticoides, en caso de dolor abdominal
• El nefroblastoma o tumor de Wilms no se debe biopsiar (sí el intenso. Si existe afectación renal intensa, se emplean inmunosu-
neuroblastoma) porque la rotura de la cápsula renal cambia presores.
el estadio. Los niños afectos de tumor de Wilms, en general,
son algo mayores que los afectos por neuroblastoma y parecen Pregunta 83.- R: 3
menos enfermos. El cuadro clínico que presenta nuestro paciente es una tos ferina,
• En cuanto a los tumores óseos, en niños mayores y adolescentes, infección por Bordetella pertussis. El periodo de incubación oscila
predomina el osteosarcoma, seguido del Ewing, que es el más entre 5-10 días. La clínica consta de dos fases:
frecuente en menores de10 años. • Fase catarral inicial: con febrícula, rinorrea y tos blanda. Es el
periodo de máxima contagiosidad.
Pregunta 81.- R: 4 • Fase de tos paroxística: consiste en accesos de tos, seguido en
El diagnóstico del VIH en el RN de madre VIH+ se ve dificultado muchas ocasiones de vómitos y acompañándose de cianosis
porque, al nacimiento, estos niños presentan Acs tipo IgG frente al facial. En los niños mayores es característico el “gallo” inspira-
VIH, no IgA (respuesta 4 falsa), procedentes de la madre que pasan torio. Esta fase puede durar 2-4 semanas. En los lactantes más
a través de la placenta. Casi todos estos niños son seropositivos al pequeños, los accesos de tos pueden acabar en pausas de apnea,
nacer y sólo un 15-30% están verdaderamente infectados. Los niños con lo cual pueden resultar letales (respuesta 1 correcta). Otras
no infectados “falsamente positivos”, pueden tardar en negativizar posibles complicaciones son: OMA, neumonías, alteraciones
anticuerpos entre 9 y 18 meses, por ello, el método de detección de de la función pulmonar de carácter transitorio (respuesta 5 co-
IgG utilizado en adultos no es un método fiable de diagnóstico en rrecta), convulsiones y hemorragias conjuntivales por esfuerzo
menores de 18 meses; hay que recurrir a procedimientos diagnósticos tusígeno (respuesta 2 correcta).
más complejos dependiendo de la edad.
La tos ferina, pese a ser infección bacteriana, cursa con linfocitosis
absoluta y en la radiografía de tórax los datos son inespecíficos (infiltra-
dos perihiliares). El diagnóstico se basa en el cultivo del moco nasal en
MENORES DE 18 MESES MAYORES DE 18 MESES
medio de Bordet-Gengou pero la detección en sangre de IgG antifactor
1. PCR-DNA + al menos en 1. Presencia de Ac frente a VIH estimulante de los linfocitos es la técnica más sensible y específica.
2 determinaciones (ELISA o Western-Blot)
2. Criterios SIDA 2. Criterios del apartado anterior Para el tratamiento se utilizan macrólidos, actualmente de elección
azitromicina pero no se emplean inmunoglobulinas (respuesta 3 falsa).
Pregunta 81. Diagnóstico de infección VIH en el niño. Debe administrarse a los pacientes y a sus familiares, para eliminar
la cadena epidemiológica de portadores, éstos suelen ser adultos
paucisintomáticos o tosedores crónicos.
Recuerda que es necesario realizar quimioprofilaxis con AZT du-
rante el embarazo, el momento del parto y luego al RN. El tratamiento Pregunta 84.- R: 4
de la infección se basa en la triple terapia, al igual que en los adultos, El cuadro clínico que se presenta es una enfermedad exantemá-
y para evaluar la evolución se emplea la carga viral y el cociente CD4/ tica con faringoamigdalitis. Se te tienen que venir, con sólo estos
CD8, recordando que la cifra de linfocitos hay que extrapolarla según datos, dos enfermedades a la mente: escarlatina y mononucleosis
la edad del niño. infecciosa. La distinción entre ambas es muy sencilla, pues el caso te
da muy buenas pistas:
Pregunta 82.- R: 4 • Petequias en paladar: muy típicas de las faringoamigdalitis
Pregunta muy fácil, sobre todo porque aunque de entrada por el estreptocócicas (escarlatina, por tanto).
enunciado no tuvieseis el diagnóstico, es fácil rechazar el resto de las • Lengua en frambuesa: a favor de la escarlatina. También se ve
respuestas, con las que no coincide el cuadro clínico descrito. en la enfermedad de Kawasaki.
• Exantema confluente en pliegues: líneas de Pastia, típicas de la
Aprovechamos para repasar las características de la púrpura escarlatina.
de Shönlein-Henoch, vasculitis propia de la infancia que afecta a • Eritema facial que respeta el triángulo nasogeniano: facies de Filatov,
pequeños vasos y esta mediada por mecanismo inmunológico de típica de la escarlatina. Recuerda a otro exantema facial: el del
predominio IgA. Se desconoce su causa, pero es frecuente que vaya eritema infeccioso, con afectación “en bofetón de ambas mejillas”.
precedida 1-2 semanas antes de una infección de vías respiratorias
altas. Posteriormente puede aparecer, en orden de frecuencia: Nos decantamos, pues, por la escarlatina:
• Afectación cutánea: púrpura palpable, en miembros inferiores, • La fiebre y la contagiosidad remiten en los primeros 1-2 días
que suele cursar en brotes con intervalos de entre unos días y tras la instauración de la antibioterapia.

CTO Medicina • C/Francisco Silvela, 106 • 28002 Madrid • Tfno.: (0034) 91 782 43 30/33/34 • E-mail: secretaria@ctomedicina.com • www.grupocto.es
16
Comentarios
Grupo CTO
CTO Medicina

• El exantema de la escarlatina a veces puede afectar a palmas


Test 2V Distancia Pediatría

• Asocia adenopatías dolorosas posteriores. Esto es muy típico


y plantas (recuerda que afectan a palmas y plantas: sarampión, de la rubéola. Pueden aparecer adenopatías posteriores en
fiebre botonosa y lúes). la mononucleosis infecciosa (si bien lo más frecuente es que
• La escarlatina secundaria a faringoamigdalitis puede complicarse sean laterocervicales bilaterales y múltiples. Recuerda que en
con glomerulonefritis y con fiebre reumática (que nunca será la mononucleosis puede aparecer exantema (por antibióticos),
secundaria a estreptococias cutáneas). esplenomegalia y faringoamigdalitis (si bien esta es exudativa,
• Las pruebas de detección rápida de antígenos de S. pyogenes y no como la de nuestro caso clínico).
el cultivo del frotis faríngeo son las pruebas de laboratorio más • Aparace un enantema: manchas rojas en el velo de paladar. Estas
útiles para el diagnóstico. son las características manchas de Forcheimer de la rubéola.
• El tratamiento de elección consiste en administrar penicilina Recuerda que las faringoamigdalitis estreptocócicas pueden
oral durante 10 días. En alérgicos se usará eritromicina, y en asociar petequias en el paladar.
pacientes de los cuales se dude el cumplimiento terapéutico,
se recurrirá a la penicilina benzatina i.m. en dosis única. De la rubéola debes recordar:
• El periodo de máxima transmisión abarca desde 7 días antes de
Pregunta 85.- R: 5 aparición del exantema hasta 7 u 8 días después de que éste
• La máxima contagiosidad en el sarampión se produce 5 días haya aparecido.
antes de la aparición de exantema. • Cursa con febrícula, conjuntivitis y adenopatías retroauriculares
• Los casos de sarampión en menores de 6 meses son raros, por y cervicales dolorosas.
la protección que ejercen los anticuerpos maternos. • El exantema es morbiliforme y confluente en cara. Se resuelve
• Las manchas de Koplik (máculas blanquecinas en la mucosa mediante una mínima descamación.
subyugal opuesta a los molares inferiores) son el enantema • Las complicaciones son poco frecuentes en la infancia: artritis
patognomónico del sarampión. (preferentemente articulaciones de pequeño tamaño), encefalitis
• En la fase prodrómica del sarampión se suele detectar, acom- y púrpura trombopénica.
pañando al cuadro catarral y a la conjuntivitis fotofóbica, una • La rubéola suele darse en niños mayores de 6 meses, pues
fiebre moderada. Ésta se eleva bruscamente cuando aparece antes los anticuerpos maternos actúan como protectores.
el exantema. Una vez superada la enfermedad suele quedar inmunidad
• El exantema es maculopapular, no pruriginoso, y sigue un curso permanente.
descendente y centrífugo. Se inicia en cara y cuello, y progresa
caudalmente hasta afectar palmas y plantas (respuesta 5 falsa). Pregunta 88.- R: 5
Se resuelve a través de una descamación furfurácea en el mismo El cuadro descrito se corresponde con una convulsión febril,
orden de aparición. La gravedad de la enfermedad se relaciona causa más frecuente de convulsión en el niño. La edad de pre-
con la intensidad y confluencia del exantema. sentación es entre los 6 meses y los 5 años de edad coincidiendo
con episodios febriles. La clínica típica consiste en fiebre alta,
Pregunta 86.- R: 2 crisis tónico-clónica generalizada de breve duración (menos de
Principales complicaciones del sarampión: 10 minutos) con un periodo postcrítico corto. Existen antece-
• Otitis media aguda. dentes familiares en un 30% de casos y no suponen un aumento
• Neumonía. Lo más frecuente es la neumonía bacteriana secun- del riesgo de padecer epilepsia en la edad adulta. La actitud más
daria (respuesta 2 falsa). La neumonía por el virus del sarampión, correcta para su manejo es el control de la temperatura corporal
o neumonía de células gigantes de Hecht, es menos frecuente, con antipiréticos, como el paracetamol. También se puede utilizar
y se da sobre todo en inmunodeprimidos. diazepam intrarrectal.
• Alteraciones neurológicas. Con gran frecuencia aparecen altera-
ciones inespecíficas en el EEG sin clínica asociada. La aparición Pregunta 89.- R: 1
de encefalomielitis (1/1.000 casos) es independiente de la seve- • La artritis por rubéola suele afectar a las pequeñas articulaciones (sobre
ridad y extensión del exantema. La panencefalitis esclerosante todo, metacarpofalángicas) de postpúberes de sexo femenino.
subaguda consiste en el deterioro del rendimiento escolar, la • La encefalitis rubeólica es 6 veces menos frecuente que la saram-
personalidad y la función intelectual; que afecta a niños en edad pionosa.
escolar con antecedente de sarampión a edades tempranas. • La profilaxis con gammaglobulina se ofrecerá a gestantes no vacu-
• El sarampión es anergizante y puede provocar la reactivación nadas que hayan contactado con el virus salvaje, como alternativa
de una tuberculosis latente. al aborto terapéutico (alto riesgo de rubéola congénita).
• Los pacientes con rubéola congénita presentan viruria mantenida
Pregunta 87.- R: 2 durante los primeros meses de vida.
En relación con las enfermedades exantemáticas, recuerda: • La probabilidad de padecer una rubéola asintomática es el doble
• El pródromos de cualquier exantemática es inespecífico, y que la de padecer la enfermedad clásica.
consiste en un cuadro catarral con fiebre.
• El exantema ya es otro cantar. En este caso es rosado, no con- Pregunta 90.- R: 3
fluente (descartamos por ello el sarampión, que es eritematoso y Esta pregunta es esencial. Ha salido muchas veces en el MIR. Re-
confluente), no pruriginoso (con lo cual descartamos la varicela). cuerda que un lactante que padece un cuadro febril de 3-4 días de
• Se acompaña de conjuntivitis (con ello descartamos la es- duración sin foco aparente, y que al solucionarse da paso a la aparición
carlatina y el exantema súbito). Los cuadros exantemáticos de un exantema maculopapuloso, poco confluente, de predominio
con conjuntivitis pueden ser debidos a: sarampión, rubéola, en tronco y miembros superiores, sólo puede ser un exantema súbito
adenovirus y enfermedad de Kawasaki. (o roséola o sexta enfermedad).

CTO Medicina • C/Francisco Silvela, 106 • 28002 Madrid • Tfno.: (0034) 91 782 43 30/33/34 • E-mail: secretaria@ctomedicina.com • www.grupocto.es
17
Pediatría
Test 2V
No debes olvidar de exantema súbito:
Distancia
Comentarios
Grupo CTO
CTO Medicina

• El tratamiento es sintomático: AINE y dieta blanda evitando la


• Generalmente está producido por el virus herpes tipo 6. ingesta de ácidos.
• Afecta principalmente a niños menores de 2 años. • Complicaciones como la pancreatitis clínica y la esterilidad
• En la fase aguda el hemograma muestra leucocitosis con neu- total residual en varones postpúberes son raras (respuestas 3 y
trofilia (como si fuese una infección bacteriana). En el periodo 5 falsas).
exantemático ya se ve el patrón típico de leucopenia con pre-
dominio de linfocitos, típico de las viriasis. Pregunta 93.- R: 2
• La complicación más frecuente, que es la crisis febril, se da en Este caso clínico presenta:
la etapa pre-exantema. También puede causar encefalitis. • A un adolescente.
• Con un proceso febril con adenopatías.
Pregunta 91.- R: 4 • Astenia.
Un exantema que no tiene pérdida es el variceloso: su carácter po- • Faringoamigdalitis exudativa.
limorfo en “cielo estrellado” (con máculas, pápulas, vesículas, pústulas • Y esplenomegalia.
y costras coexistiendo sobre la piel del paciente) que suele afectar a
tronco y parte proximal de extremidades. La afectación de mucosas Este es el “retrato robot” de la mononucleosis infecciosa, cuyo
(oral, anal, genital) y su cualidad pruriginosa lo hacen reconocible. La agente etiológico es el virus de Epstein-Barr (respuesta 1 correcta).
opción correcta, por tanto, es la 4. Recuerda que hay síndromes mononucleósicos producidos por otros
gérmenes (CMV, que cursa con faringoamigdalitis no exudativa y
Recuerda que la fase prodrómica consiste en un catarro de vías adenopatías de pequeño tamaño; Toxoplasma gondii, que cursa
altas con febrícula. La complicación más frecuente es la sobreinfección sin faringitis; rubéola; HIV). El VEB tiene un claro tropismo por los
bacteriana de las lesiones. Otras complicaciones son: la neumonía va- linfocitos B (respuesta 2 falsa). Recuerda que los clásicos linfocitos
ricelosa (en la infancia es más frecuente la secundaria a sobreinfección atípicos de la mononucleosis son linfocitos T activados para destruir
bacteriana, más que la producida por el propio virus) y la afectación a los linfocitos B colonizados por el virus.
del SNC (lo más frecuente es en forma de cerebelitis, que evoluciona
de forma favorable). El VEB provoca en gran número de ocasiones una hepatitis sub-
clínica, con lo cual se acompaña de elevación de las transaminasas
La varicela en los niños sanos no precisa tratamiento con aciclovir. (respuesta 3 correcta).
Son indicaciones para su uso: neonatos e inmunodeprimidos. Si a un paciente con mononucleosis se le trata con betalactámi-
Opción 1. La primoinfección herpética (por herpes I) sólo afecta cos, aparecerá un exantema inespecífico en un 80% de las ocasiones
a la mucosa oral, en forma de gingivoestomatitis. (respuesta 4 correcta).

Opción 2. La enfermedad mano-pie-boca se da por enterovirus, La determinación de anticuerpos heterófilos puede ayudar al
y cursa con la aparición de vesículas (sólo vesículas) en los territorios diagnóstico de esta enfermedad (respuesta 5 correcta), si bien el
referidos. dato analítico más específico es la determinación serológica de la
IgM anti-VEB.
Opción 3. La erupción variceliforme de Kaposi produce una afec-
tación generalizada secundariamente a una infección por virus herpes, Pregunta 94.- R: 5
y afecta a inmunodeprimidos o pacientes con dermatitis atópica (la Recuerda las diferencias más significativas del SIDA infantil respecto
paciente del enunciado es una niña sana). al del adulto.
• El periodo de incubación es más corto en los niños (respuesta
Opción 5. La herpangina cursa con vesículas en los pilares ante- 1 falsa). Se subdivide en SIDA precoz (periodo de latencia de
riores de la faringe. pocos meses, clínica antes del año de vida, predominando las
manifestaciones neurológicas, pronóstico malo) y SIDA tardío
Pregunta 92.- R: 3 (más parecido al del adulto, predominando la clínica infecciosa).
Los datos clínicos de dolor, tumefacción que borra el ángulo de la • La hipergammaglobulinemia es más precoz (respuesta 2 falsa).
mandíbula y fiebre en un niño, deben hacernos pensar en una parotiditis. • El número de infecciones bacterianas y de alteraciones neuroló-
gicas es mayor que en los adultos (respuesta 3 falsa). Las manifes-
De la parotiditis debes recordar: taciones clínicas más frecuentes en pediatría, comparándolo con
• Hasta el 40% de casos de parotiditis cursan de forma subclínica los adultos son: cuadros ORL, parotiditis, neumonía intersticial
(respuesta 1 falsa). linfoide y manifestaciones neurológicas (calcificaciones de los
• Si da clínica, se observa edema localizado entre rama ascen- ganglios basales, atrofia cerebral...). Sin embargo, las infecciones
dente de mandíbula y mastoides que desplaza lóbulo de la oportunistas son menos frecuentes que en adultos (respuesta
oreja hacia arriba y hacia afuera. En el 75% de casos es bilateral 5 correcta) pero de mayor agresividad. También son menos
pero de evolución asimétrica. La piel suprayacente a la glándula frecuentes la hepatitis B, los linfomas y el sarcoma de Kaposi.
inflamada no está afectada, y la palpación suele ser dolorosa La causa más frecuente de muerte es la neumonía por P. carinii.
(respuesta 4 falsa). • La inversión del cociente CD4/CD8 es más tardía y la linfopenia
• Las complicaciones pueden aparecer en ausencia de enferme- es menor (respuesta 4 falsa).
dad manifiesta, siendo la más frecuente la meningitis aséptica
(urliana); hasta en el 65% de los pacientes se detecta pleocitosis Pregunta 95.- R: 3
en el LCR, aunque no presenten clínica de meningoencefalitis El SMSL, muerte repentina e inexplicable de un niño menor de
(respuesta 2 cierta). un año, supone la causa principal de muerte entre el mes de vida y

CTO Medicina • C/Francisco Silvela, 106 • 28002 Madrid • Tfno.: (0034) 91 782 43 30/33/34 • E-mail: secretaria@ctomedicina.com • www.grupocto.es
18
Comentarios
Grupo CTO
CTO Medicina

el año de edad en los países desarrollados. Este síndrome se correla-


Test 2V Distancia Pediatría

a los padres sobre síntomas que anuncien una infección (tos, rino-
ciona con la posición de decúbito prono y lateral para dormir (opción rrea, vómitos, diarrea) porque la causa más frecuente de convulsión
3 incorrecta). Tras este factor posicional, el tabaquismo materno es entre los 6 meses y los 5 años son las crisis febriles. Hasta un 5% de
actualmente el principal factor de riesgo del SMSL. Otros factores lactantes las padece.
son la prematuridad, el antecedente de hermano muerto por SMSL,
sexo masculino, etc. Debido a la ausencia de tratamiento es muy Las crisis febriles típicas duran menos de 10 minutos y son genera-
importante la prevención con medidas como la posición de decúbito lizadas. Ante síntomas de focalidad, habrá que hacer una exploración
supino para dormir y la lactancia materna (que se considera el factor neurológica más detallada, que incluirá en todos los casos EEG.
preventivo más importante).
La complicación más frecuente de las crisis febriles es la re-
Pregunta 96.- R: 3 currencia, en un 40-50% de los casos. No suponen un aumento
La tetralogía de Fallot es un complejo malformativo formado por: del riesgo de padecer epilepsia en la edad adulta. La actitud más
• Comunicación interventricular. correcta para su manejo es el control de la temperatura corporal
• Estenosis pulmonar. con antipiréticos como el paracetamol. También se puede utilizar
• Acabalgamiento de la aorta sobre el defecto septal ventricular. diazepam intrarrectal.
• Hipertrofia del ventrículo derecho.
Pregunta 99.- R: 3
Es la cardiopatía cianosante más frecuente a partir del año de Descripción de “retrato robot” de un cuadro epiléptico propio de
edad, que cursa con disnea que clásicamente mejora con la posición la infancia: el síndrome de West.
de cuclillas. El hecho determinante de la clínica de la tetralogía de • Etiología: puede ser idiopático o secundario a lesión neurológica
Fallot es el grado de obstrucción a la salida del ventrículo derecho. (esclerosis tuberosa, anoxia perinatal, infecciones del SNC).
La disminución de las resistencias periféricas por vasodilatación (ejer- • Epidemiología: lactante (inicio entre el 4º-9º mes).
cicio, llanto…), aumentan el shunt derecha-izquierda provocando • Clínica: comienza con espasmos (los más frecuentes son los
crisis hipoxémicas. mixtos, con flexión y extensión de miembros). Varios días
después hay detención del desarrollo y desconexión del
Al haber obstrucción a la salida del ventrículo derecho, en el ECG medio.
veremos signos de hipertrofia compensatoria del mismo. La altera- • Trazado electroencefalográfico: muestra un patrón caracterís-
ción radiológica del Fallot es el “corazón en zueco”, con el ápex tico llamado hipsarritmia (para que lo recuerdes, es un patrón
levantado, la silueta ventricular derecha aumentada de tamaño y los caótico con actividad basal desorganizada, potenciales amplios,
pulmones hiporémicos. Recuerda que se ausculta un soplo sistólico con espigas, polipuntas-ondas, etc.).
en foco pulmonar. • El tratamiento de primera elección del síndrome de West es el
ACTH; de segunda elección se recurre al ácido valproico. En
Pregunta 97.- R: 2 casos rebeldes se pueden emplear combinaciones terapéuticas
Es importante que recuerdes la radiología típica de la coarta- o los nuevos anticomiciales.
ción aórtica. Se aprecian muescas costales en los bordes inferiores
de las costillas (signo de Rösler), debidas al engrosamiento de las Pregunta 100.- R: 5
arterias costales por el aumento de la presión arterial y, a veces, En relación con el trastorno por hiperactividad con déficit de
el llamado signo del 3 o escotadura de la aorta en el lugar de la atención recuerda:
coartación. • Es la entidad psiquiátrica más frecuente en la infancia. Más
frecuente en varones 3/1.
La CoA consiste en un estrechamiento de la luz aórtica. Recuerda • Parece haber un componente heredofamiliar importante y tiene
que es más frecuente en niñas con síndrome de Turner. El tipo más mayor prevalencia en medios sociales desfavorecidos.
frecuente es yuxtaductal. • Se ha demostrado la asociación entre el gen del receptor do-
paminérgico D4 y el fenotipo de THDA.
Los niños suelen estar asintomáticos y la enfermedad puede pasar • Clínica: escasa capacidad de atención con hiperactividad
desapercibida hasta la edad adulta. En la exploración es típico en- motora, inquietud, labilidad emocional, baja tolerancia a la
contrar pulsos en extremidades inferiores disminuidos y retrasados en frustración, etc. Suele mejorar con la pubertad pero con fre-
comparación con los miembros superiores, lo que supone HTA en cuencia evoluciona a fracaso escolar, consumo de drogas, baja
miembros superiores. En el ECG aparecen signos de hipertrofia de VI. autoestima, depresión y ansiedad entre otros.
• Hay pruebas de imagen que han documentado patología
El tratamiento es quirúrgico en niños. estructural en los niños con THDA en el circuito prefrontal-
estriado-tálamo-corteza.
Pregunta 98.- R: 4 • Para su tratamiento no se emplean sedantes (respuesta 5 falsa),
Cuando a un Servicio de Urgencias pediátricas llega un niño pe- sino estimulantes (metilfenidato) para mejorar la atención y
queño convulsionando, hay que tomarle la temperatura e interrogar psicoterapia.

CTO Medicina • C/Francisco Silvela, 106 • 28002 Madrid • Tfno.: (0034) 91 782 43 30/33/34 • E-mail: secretaria@ctomedicina.com • www.grupocto.es
19

S-ar putea să vă placă și